física moderna capítulo 4 - radiação de corpo negro e ... · de correntes de convecção cde ar...

40
Física Moderna Capítulo 4 - Radiação de corpo negro e Efeito Fotoelétrico Prof. Dr. Cláudio S. Sartori 23 23 x z E B Radiação. Lei de Stefan-Boltzmann. Corpo Negro e Radiador Ideal. Radiação Quando se coloca uma das mãos em contato direto com a superfície de um aquecedor d'água ou radiador a vapor, o calor a atinge por condução através das paredes do radiador. Se a mão for colocada acima do mesmo, mas sem tocá-lo, o calor a atinge por meio de correntes de convecção de ar aquecido, que se movem para cima. Colocando-se em um dos lados do radiador, ela ainda se torna quente, embora a condução através do ar seja desprezível e esteja fora da trajetória das correntes de convecção. A energia térmica, agora, é transmitida por radiação. O termo radiação refere-se á emissão contínua de energia da superfície de todos os corpos. É chamada energia radiante e tem a forma de ondas eletromagnéticas . Essas ondas propagam-se com a velocidade da luz e são transmitidas através do vácuo ou do ar. (Na realidade, transmitem-se melhor no vácuo, pois no ar são parcialmente absorvidas.) Quando atingem um corpo que não lhes é transparente como, por exemplo, a superfície da mão ou as paredes de um quarto, são absorvidas. Figura 1 Ondas eletromagnéticas. Figura 2 Variação dos comprimentos de onda . c h f h E Figura 3 Variação do comprimento de onda, freqüência e energia para ondas eletromagnéticas. h é a constante de Planck: 34 6.62608 10 h Js Figura 4,5,6 Ampliação da variação do comprimento de onda, para ondas eletromagnéticas a) e b) e Radiância espectral c): a) Ondas de Rádio: b) Espectro visível. c) Radiância Espectral indicando as curvas dos resultados obtidos pelos os modelos de Planck (atual) e Rayleigh-Jeans No início do século, Rayleigh, e também Jeans, fizeram o cálculo da densidade de energia clássica da

Upload: vukien

Post on 16-Dec-2018

216 views

Category:

Documents


0 download

TRANSCRIPT

Page 1: Física Moderna Capítulo 4 - Radiação de corpo negro e ... · de correntes de convecção cde ar aquecido, que se movem para cima. Colocando-se em um dos lados do ... para a densidade

Física Moderna – Capítulo 4 - Radiação de corpo negro e Efeito Fotoelétrico

Prof. Dr. Cláudio S. Sartori

23

23

x

y

zE

B

Radiação. Lei de Stefan-Boltzmann. Corpo

Negro e Radiador Ideal.

Radiação

Quando se coloca uma das mãos em contato

direto com a superfície de um aquecedor d'água ou

radiador a vapor, o calor a atinge por condução através

das paredes do radiador. Se a mão for colocada acima

do mesmo, mas sem tocá-lo, o calor a atinge por meio

de correntes de convecção de ar aquecido, que se

movem para cima. Colocando-se em um dos lados do

radiador, ela ainda se torna quente, embora a condução

através do ar seja desprezível e esteja fora da trajetória

das correntes de convecção. A energia térmica, agora,

é transmitida por radiação.

O termo radiação refere-se á emissão contínua de

energia da superfície de todos os corpos. É chamada

energia radiante e tem a forma de ondas

eletromagnéticas. Essas ondas propagam-se com a

velocidade da luz e são transmitidas através do vácuo

ou do ar. (Na realidade, transmitem-se melhor no

vácuo, pois no ar são parcialmente absorvidas.)

Quando atingem um corpo que não lhes é transparente

como, por exemplo, a superfície da mão ou as paredes

de um quarto, são absorvidas.

Figura 1 – Ondas eletromagnéticas.

Figura 2 – Variação dos comprimentos de onda .

chfhE

Figura 3 – Variação do comprimento de

onda, freqüência e energia para ondas

eletromagnéticas.

h é a constante de Planck:

346.62608 10h J s

Figura 4,5,6 – Ampliação da variação do

comprimento de onda, para ondas eletromagnéticas

a) e b) e Radiância espectral c):

a) Ondas de Rádio:

b) Espectro visível.

c) Radiância Espectral indicando as curvas dos

resultados obtidos pelos os modelos de Planck (atual)

e Rayleigh-Jeans

No início do século, Rayleigh, e também Jeans,

fizeram o cálculo da densidade de energia clássica da

Page 2: Física Moderna Capítulo 4 - Radiação de corpo negro e ... · de correntes de convecção cde ar aquecido, que se movem para cima. Colocando-se em um dos lados do ... para a densidade

Física Moderna – Capítulo 4 - Radiação de corpo negro e Efeito Fotoelétrico

Prof. Dr. Cláudio S. Sartori

24

24

radiação de cavidade. Podemos considerar uma

cavidade cúbica com paredes metálicas aquecidas

uniformemente à temperatura T contendo radiação

eletromagnética, formando ondas estacionárias nas

paredes da caixa. As paredes emitem radiação

eletromagnética na faixa térmica de freqüência.

Também denominamos esse modelo de corpo negro.

Esse cálculo mostrou uma séria divergência entre a

física clássica e os resultados experimentais.

A fórmula de Rayleigh-Jeans para o corpo

negro é dada por:

dc

kTdT 3

28)(

: Freqüência da radiação

: densidade de energia

k: Constante de Boltzman: KJk 231038,1

c: velocidade da luz: smc 8100,3

A função de distribuição de Boltzmann usada é

dada por:

k Te

Pk T

e o valor médio das energias do sistema:

0

0

P d

k T

P d

Ao tentar solucionar essa discrepância entre a

teoria e a experiência, Planck foi levado a considerar a

hipótese de uma violação da lei da equipartição da

energia, sobre a qual a teoria clássica se baseava.

Planck supôs que a energia poderia ter apenas

alguns valores discretos, em vez de qualquer valor, e

que os valores discretos fossem uniformemente

distribuídos, da forma:

0, ,2 ,3 ,4 ,

Assim:

0

0

n

n

P

P

0

0

n

n

k Tn

n

k T

n

e

e

k T

Como n n h

0

0

n h

k T

n

n h

k T

n

n he

k T

e

k T

Chamando: h

k T

0

0

n

n

n

n

n e

k T

e

Note que:

0

0

0

ln

n

n n

nn

n

de

d de

de

0

0

0

ln

n

n n

nn

n

n ed

ed

e

Logo, podemos deduzir que::

0

0

0

ln

n

n n

nn

n

n ed

ed

e

Assim:

0

0

n

n

n

n

n e

k T

e

Page 3: Física Moderna Capítulo 4 - Radiação de corpo negro e ... · de correntes de convecção cde ar aquecido, que se movem para cima. Colocando-se em um dos lados do ... para a densidade

Física Moderna – Capítulo 4 - Radiação de corpo negro e Efeito Fotoelétrico

Prof. Dr. Cláudio S. Sartori

25

25

x

y

zE

B

0

0

n

n

n

n

n e

k T

e

0

ln n

n

dk T e

d

0

ln n

n

h dk T e

k T d

0

ln n

n

dh e

d

2 3

0

1n

n

e e e e

2 3

0

1n

n

e x x x x e

2 3 11

1x x x

x

0

1

1

n

n

ee

1

0

1n

n

e e

1

ln 1d

h ed

1 ln 1d

h ed

0 1

1

eh

e

1

eh

e

1

e eh

e e

1

1h

e

1h

k T

h

e

Figura 7 – (a) Modelo teórico da radiação de

corpo negro de Planck e dados experimentais.

Assim, Planck utilizou a fórmula que ele obteve

para a densidade de energia do espectro do corpo

negro, considerando modificações importantes na

distribuição clássica feita por Boltzmann; seu

resultado para a distribuição de energia foi dado por:

1

Tk

h

e

hE

Aqui h é a chamada constante de Planck e vale:

sJh 341063,6 .

Define-se também:2

h

A fórmula para a densidade de energia do

espectro do corpo negro, utilizando essa distribuição

de energia foi:

d

e

h

cd

Tk

hT

1

83

2

Esse é o espectro de corpo negro de Planck.

Se fizermos o espectro para comprimentos de

onda, teremos:

dc

dc

2

d

e

hcd

Tk

chT

1

185

Page 4: Física Moderna Capítulo 4 - Radiação de corpo negro e ... · de correntes de convecção cde ar aquecido, que se movem para cima. Colocando-se em um dos lados do ... para a densidade

Física Moderna – Capítulo 4 - Radiação de corpo negro e Efeito Fotoelétrico

Prof. Dr. Cláudio S. Sartori

26

26

11014

21014

31014

41014

51014

61014

510-17

110-16

1.5 10-16

210-16

2.5 10-16

310-16

Figura 7 – Densidade de energia para diferentes

temperaturas, em função:

(b) Do comprimento de onda . Observe que

o pico do máximo desloca-se para a esquerda a medida

que a temperatura aumenta.

(c) Da freqüência . Observe o deslocamento

do pico para a direita conforme o aumento da

temperatura.

A energia radiante emitida por uma superfície,

por unidade de tempo e de área, depende da natureza e

da temperatura do corpo. A baixas temperaturas, a taxa

de radiação é pequena e a energia radiante consiste

principalmente em comprimentos de onda

relativamente longos. À medida que a temperatura

aumenta, a taxa de radiação cresce rapidamente, sendo

diretamente proporcional à quarta potência da

temperatura absoluta. Por exemplo, um bloco de cobre

à temperatura de 100°C (373 K) irradia cerca de 0,03 J

• s -1

ou 0,03 W por cm2 de sua superfície, enquanto a

500°C (773 K), sua radiação é de 0,54 W por cm2.

Já a l 000°C (l 273 K), ela irradia cerca de 4

W por cm2. Essa taxa de radiação é cerca de 130 vezes

maior do que a uma temperatura de 100°C.

Em qualquer temperatura, a energia radiante

emitida é uma mistura de ondas de comprimento de

onda diferentes. Comprimentos de onda na faixa do

espectro visível variam de 0.4 . 10-6

m (violeta) até 0.7

.10 -6

m (vermelho). Na temperatura de 300°C, quase

toda a energia radiante emitida por um corpo tem

comprimentos de onda maiores do que esses. Tais

ondas são chamadas infravermelhas. Quando a

temperatura aumenta, os comprimentos de onda

desviam-se para valores menores. A 800°C, um

corpo emite bastante energia visível para ser

luminoso e aparece avermelhado. Ainda assim, a

maior parte da energia radiante ainda está no

infravermelho. A 3 000°C, que corresponde

aproximadamente à temperatura do filamento de uma

lâmpada incandescente, a energia radiante contém

uma proporção suficiente dos comprimentos de onda

mais curtos para parecer brancos.

Um pirômetro óptico é um dispositivo que

mede temperatura sem contacto com o corpo do qual

se pretende conhecer a temperatura. Geralmente este

termo é aplicado a instrumentos que medem

temperaturas superiores a 6000C. Uma utilização

típica é a medição da temperatura de metais

incandescentes em fundições.

Um dos pirómetros mais comuns é o de

absorção-emissão, que é utilizado para determinar a

temperatura de gases através da medição da radiação

emitida por uma fonte de referência, antes e depois

da radiação incidir sobre o gás (que absorve parte da

radiação). É através da análise das diferenças do

espectro do gás que se consegue determinar a sua

temperatura. Ambas as medições são feitas no

mesmo intervalo de comprimentos de onda.

Outra aplicação típica do pirómetro é a medição

da temperatura de metais incandescentes. Olhando

pelo visor do pirômetro observa-se o metal,

ajustando-se depois manualmente a corrente eléctrica

que percorre um filamento que está no interior do

pirómetro e aparece no visor. Quando a cor do

filamento é idêntica à do metal, pode-se ler a

temperatura numa escala disposta junto ao elemento

de ajuste da cor do filamento.

A seguir indicamos o espectro solar obtido

experimentalmente, e veja a concordância com o

modelo da radiação de corpo negro de Planck.

Figura 8 - Radiância espectral solar e absorção

atmosférica. Observe o modelo da radiância do

corpo negro pontilhado em vermelho.

Page 5: Física Moderna Capítulo 4 - Radiação de corpo negro e ... · de correntes de convecção cde ar aquecido, que se movem para cima. Colocando-se em um dos lados do ... para a densidade

Física Moderna – Capítulo 4 - Radiação de corpo negro e Efeito Fotoelétrico

Prof. Dr. Cláudio S. Sartori

27

27

2.5 10-7510

-77.5 10

-7110

-61.25 10

-61.5 10

-61.75 10

-6210

-6Wavelength , m

21013

41013

61013

81013

11014

Power , Wm^2

4000. K

5000. K

6000. K

Lei do Deslocamento de Wien

Ao considerarmos a função de distribuição em

termos do comprimento de onda :

d

e

hcd

Tk

chT

1

185

Aqui h é a chamada constante de Planck e vale:

sJh 341063,6 : Comprimento de onda da radiação

: densidade de energia.

k: Constante de Boltzmann:

KJk 231038,1

c: velocidade da luz: 83,0 10 m

sc

Derivando em relação a :

5

8 1

1

T

h c

k T

hc

e

Igualando a derivada a zero para encontrarmos

em qual comprimento de onda ocorrerá o máximo de

radiação, chega-se a:

1 05

h c h c

k T k Th c

e ek T

Chamando de: h c

xk T

Geramos uma equação para f(x): ( ) 1 0

5

x xf x e

Aplicando métodos numéricos para a solução

desta equação, chega-se ao valor:

4,96511423175275x

h cx

k T

34 8

23

6.63 10 3.0 104,96511423175275

1,38 10 T

32.898 10T m

Esta é conhecida como Lei do deslocamento

de Wien, onde o comprimento de onda da radiação,

, está em m.

Podemos também considerar o comprimento

de onda da radiação em mm:

2.898T mm

Lei de Stefan-Boltzmann

A experiência mostra que a taxa de radiação

da energia por uma superfície é proporcional à área

da superfície e à quarta potência da temperatura

absoluta T. Depende também da natureza da

superfície, descrita por um número adimensional e,

que está entre 0 e l. Assim, a relação pode ser

expressa por:

4

0

TdRH T

4H A e T

onde ( é uma constante universal da Física,

chamada constante de Stefan-Boltzmann. Esta

relação foi deduzida por Josef Stefan (1835-1893)

com base nos resultados experimentais feitos por

John Tyndall (1820-1893) e, posteriormente,

derivada por considerações teóricas por Ludwig

Boltzmann (1844-1906).

A radiação de cavidade H é proporcional à

densidade de energia ·:

)()( TTR

Figura 9 - Radiância espectral para

diversas temperaturas, mostrando o deslocamento

em , para a esquerda (indo para a região do UV), à

medida em que a temperatura aumenta.

Page 6: Física Moderna Capítulo 4 - Radiação de corpo negro e ... · de correntes de convecção cde ar aquecido, que se movem para cima. Colocando-se em um dos lados do ... para a densidade

Física Moderna – Capítulo 4 - Radiação de corpo negro e Efeito Fotoelétrico

Prof. Dr. Cláudio S. Sartori

28

28

Na equação anterior, H tem unidades de

potência (energia por unidade de tempo). Assim, no

SI, o tem unidades de W.m-2

K-4

. O valor numérico de

σ é:

8

4 25.6699 10

W

K m

O número e, que caracteriza as propriedades

de emissão de uma dada superfície, é chamado

emissividade. Em geral, ele é maior para superfícies

escuras e ásperas do que para superfícies lisas e claras.

A emissividade de uma superfície polida de cobre é

aproximadamente de 0,3.

Exemplo 3 - Uma fina placa quadrada de aço,

com 10 cm de lado, é aquecida até a temperatura de

800°C. Sendo a emissividade igual a l, qual a taxa

total de radiação de energia?

Solução. A área total, incluindo ambos os lados

é 2 (0,1 m)2 = 0,02 m

2. A temperatura que deve ser

colocada na anterior tem de ser a temperatura

absoluta, isto é, 800°C = l 073 K. A equação dá, então,

H = (0,02 m2) (l) (5,67 10

-8 Wm

-2K

-4 ) (l 073 K)

4

H = l 503 W.

Se a placa fosse aquecida por meio de um

aquecedor elétrico, a potência de l 503 W teria que ser

fornecida para manter a sua temperatura constante e

igual a 800°C.

Se a superfície de qualquer corpo estiver

continuamente emitindo energia radiante por que,

eventualmente, não irradia toda sua energia interna e

resfria-se até a temperatura do zero absoluto (onde H =

0, pela equação). A resposta é que assim aconteceria

se, de certa maneira, não fosse fornecida energia ao

mesmo. No caso do filamento de uma lâmpada

elétrica, a energia é fornecida eletricamente para

compensar a energia radiada. Logo que se corta o

fornecimento de energia ao mesmo, ele se resfria

rapidamente até atingir a temperatura ambiente. A

temperatura não desce mais porque o ambiente (as

paredes e outros objetos no quarto) também está

radiando e certa quantidade desta energia radiante é

interceptada, absorvida e convertida em energia

interna. A mesma coisa é válida para todos os outros

objetos no quarto — estão simultaneamente emitindo e

absorvendo energia. Se qualquer objeto estiver mais

quente que o ambiente, sua taxa de emissão excederá

a de absorção. Haverá, assim, uma perda efetiva de

energia e o corpo se resfriará, a menos que seja

aquecido por um outro processo. Se, ao contrário, a

temperatura do corpo for mais baixa que a do

ambiente, sua taxa de absorção será maior que a de

emissão e a temperatura elevar-se-á. Quando o corpo

tiver a mesma temperatura que o ambiente, as duas

taxas tornar-se-ão iguais, não haverá perda ou ganho

de energia e a temperatura não variará.

Se um pequeno corpo de emissividade e

estiver completamente envolvido por paredes cuja

temperatura é T, a taxa de absorção de energia

radiante, por unidade de área, pelo corpo será:

H = Aea T4.

Daí, para um tal corpo a uma temperatura

T1 e envolvido por paredes cuja temperatura é T2, a

taxa efetiva de perda (ou ganho) de energia, por

unidade de área, por radiação, é:

4 4

1 2efH A e T T

A emissão infravermelha de um corpo pode

ser estudada por meio de uma câmara equipada com

filme sensível ao infravermelho, ou com um aparelho

semelhante, em princípio, a uma câmara de televisão

e sensível à radiação infravermelha. A fotografia

resultante é chamada termografïa.

Uma vez que a emissão depende da

temperatura, a termografïa permite o estudo

detalhado das distribuições de temperatura. Alguns

instrumentos atualmente são sensíveis a diferenças de

temperatura de até 0,1°C.

A termografïa tem uma grande variedade de

aplicações médicas importantes. Variações locais de

temperatura no corpo estão associadas a vários tipos

de tumores, como câncer no seio e distúrbios no

diâmetro de vasos, até um centímetro, podem ser

detectados. Distúrbios vasculares que geram

anomalias locais de temperatura podem ser estudados

e muitos outros exemplos poderiam ser citados.

Radiador Ideal Imagine que as paredes de um recipiente

fechado sejam mantidas à temperatura T; e que vários

corpos de diferentes emissividades sejam suspensos

sucessivamente dentro do recipiente.

Independentemente das temperaturas dos corpos que

são introduzidos, vê-se que, eventualmente, cada um

atinge a mesma temperatura Ti, isto é, os corpos

atingem o equilíbrio térmico com o ambiente.

Quando em equilíbrio térmico, o corpo emite energia

radiante. Parte desta energia é refletida e a restante,

absorvida. Na ausência outro processo qualquer, a

Page 7: Física Moderna Capítulo 4 - Radiação de corpo negro e ... · de correntes de convecção cde ar aquecido, que se movem para cima. Colocando-se em um dos lados do ... para a densidade

Física Moderna – Capítulo 4 - Radiação de corpo negro e Efeito Fotoelétrico

Prof. Dr. Cláudio S. Sartori

29

29

energia absorvida elevará a temperatura do corpo

absorvente, mas como se observa que a temperatura

não varia, cada corpo deve emitir energia radiante na

mesma proporção que a absorve, Assim, um bom

absorvente é um bom emissor e um mal absorvente,

um mal emissor.

Mas como cada corpo deve absorver ou

refletir a energia radiante que o atinge, um mal

absorvente deve ser também um bom refletor. Assim,

um bom refletor é um mal emissor.

Esta é a razão das paredes das garrafas

térmicas serem prateadas. Tais recipientes são

fabricados com paredes duplas de vidro, entre as quais

se faz vácuo, de tal maneira que os fluxos de calor por

convecção condução são praticamente eliminados. A

fim de reduzir ao máximo as perdas por radiação,

cobrem-se a paredes com uma camada de prata, que é

altamente refletora e, portanto, muito má emissora.

Como um bom absorvente é um bom emissor,

o melhor emissor será aquele cuja superfície for mais

absorvente. Mas nenhuma superfície poderá absorver

maior quantidade de energia radiante do que a que

incide sobre ela. Qualquer superfície que absorve toda

a energia incidente será a melhor emissora possível

não refletiria energia radiante e apareceria, então, com

a cor negra (contanto que sua temperatura não seja tão

alta a torná-la autoluminosa) e, por isso, chama-se

superfície negra ideal; um corpo possuidor de tal

superfície é denominado corpo negro ideal, radiador

ideal ou simplesmente corpo negro.

Nenhuma superfície real é idealmente negra;

a mais aproximada é o negro-de-fumo, que reflete

apenas cerca de 1%, Entretanto, podem-se quase obter

as condições ideais de um corpo negro, fazendo-se

uma pequena abertura nas paredes de um recipiente

fechado. A energia radiante que entrar na abertura será

parcialmente absorvida pelas paredes interiores. Da

parte refletida, apenas uma quantidade muito pequena

escapa pela abertura, a restante sendo eventualmente

absorvida pelas paredes. Daí a abertura comportar-se

como um absorvente ideal.

Inversamente, a energia radiante emitida pelas

paredes ou por qualquer corpo dentro do recipiente

que escapa pela abertura, terá a mesma natureza que a

emitida por um radiador ideal, desde que as paredes

tenham uma temperatura uniforme. Esse fato é

importante quando se usa um pirômetro óptico. As

leituras de tal instrumento só serão corretas quando ele

for dirigido para um corpo negro. Se usado para medir

a temperatura de um lingote de ferro aquecido ao

rubro, ao ar livre, suas leituras seriam muito baixas,

pois o ferro é um emissor pior que o corpo negro. Se,

entretanto, o pirômetro for dirigido par o ferro

enquanto estiver ainda na fornalha, onde está

circundada por paredes à mesma temperatura, a

―condição de corpo negro‖ serão preenchidas e a

leitura será correia. A falha do ferro em emitir tão

efetiva mente quanto um corpo negro será justamente

compensada pela energia radiante que ele reflete.

A emissividade e de uma superfície

idealmente negra é igual à unidade. Para qualquer

superfície real à uma fração menor que l.

Exemplo 4 - Exemplo. Supondo que a

superfície total do corpo humano tenha l,2 m2 e que a

temperatura da superfície seja de 30°C = 303 K,

achar a taxa total de radiação de energia pelo corpo.

Solução. Surpreendentemente, para a radiação

infravermelha o corpo humano é uma ótima

aproximação de um corpo negro ideal,

independentemente de pigmentação da pele. A taxa

da perda de energia é dada, usando e = l:

H = (1,2 m2) (l) (5,67 10

-8 W • m-

2 • K

-4) (303

K)4 = 574 W.

Obviamente, esta perda é parcialmente

balanceada pela absorção de radiação, que depende

da temperatura do ambiente. A taxa líquida de

transferência de energia radiante é dada pela equação

anterior.

Page 8: Física Moderna Capítulo 4 - Radiação de corpo negro e ... · de correntes de convecção cde ar aquecido, que se movem para cima. Colocando-se em um dos lados do ... para a densidade

Física Moderna – Capítulo 4 - Radiação de corpo negro e Efeito Fotoelétrico

Prof. Dr. Cláudio S. Sartori

30

30

Características da Radiação

Eletromagnética e suas aplicações:

Faixa

(metros/hertz)

Variação

Específica

Ondas de Rádio

c

f

cf

104 - 10-2 m/104 -

1010 Hz c f

ultra-low frequency

(ULF) 3 - 30 Hz

extremely low frequency (ELF)

30 - 300 Hz

voice frequencies (VF)

300 Hz - 3 kHz

very low frequency

(VLF)

3 - 30 kHz

low frequency (LF) 30 - 300 kHz

medium frequency

(MF)

300 kHz - 3 MHz

high frequency

(HF)

3 - 30 MHz

very high frequency

(VHF)

30 - 300 MHz

ultra high

frequency (UHF)

300 MHz - 3

GHz

super high frequency (SHF)

3 - 30 GHz

extremely high frequency (EHF)

30 - 300 GHz

shortwave see MF, HF

television see VHF, UHF

microwave 30 cm - 1 mm/1-300 GHz

Infrared - Infravermelho

10-3 - 10-6 m/1011 -

1014 Hz = 10-6

n = 10-9

far 1000-30 m

middle 30-3 m

near 3-0.75 m

Espectro visível Visible

5x10-7 m/2x1014 Hz

Red – Vermelho 770-622 nm

Orange – Laranja 622-597 nm

Yellow – Amarelo 597-577 nm

Green – Verde 577-492 nm

Blue – Azul 492-455 nm

Violet – Violeta 455-390 nm

Ultraviolet –

Ultravioleta

10-7 - 10-8 m/1015 -

1016 Hz

UV-A (least

harmful)

400-315 nm

UV-B (more

harmful, absorbed by ozone)

315-280 nm

UV-C (most

harmful, but all

absorbed by air)

280-100 nm

near UV ("black

light")

400-300 nm

far UV 300-200 nm

vacuum UV 200-100 nm

X ray

Raio X

10-9 - 10-11 m/1017 -

1019 Hz

Gamma ray

Raios gama

10-11 - 10-13 m/1019 -

1021 Hz

Submúltiplos

do

comprimento

de onda

Notação

Valor

Nanômetro nm 1 nm = 10-9

m micrômetro µm 1 µm = 10

-6 m

Angstron 1 Å 1 Å = 10-10m

Espectro Eletromagnético de Radiação:

Região 0

A

cm

F

Hz

E

eV

Radio Rádio > 109 > 10 < 3.109 < 10-5

Microwave-

Microonda

109 -

106 10 - 0.01

3.109 –

3.1012

10-5 -

0.01

Infrared-

Infravermelho

106 -

7000

0.01 - 7

10-5

3.1012 -

4.3.1014 0.01 - 2

Visible-

Visível

7000 -

4000

7.10-5 –

4.10-5

4.3.1014 -

7.5.1014 2 - 3

Ultraviolet-Ultravioleta

4000 - 10

4.10-5 - 10-7

7.5.1014 – 3.1017

3 - 103

X-Rays-Raio X

10 - 0.1

10-7 - 10-

9 3.1017 – 3.1019

103 - 105

Gamma

Rays-Raios

Gama

< 0.1 < 10-9 > 3.1019 > 105

Ondas de rádio têm os comprimentos de

onda mais longos do espectro eletromagnético. Estas

ondas podem ser mais longas que um campo de

futebol ou tão pequeno quanto uma bola de futebol.

Ondas de rádio fazem mais do que trazer música para

seu rádio. Eles também levam sinais para sua

televisão e telefones celulares. As antenas fixadas em

sua televisão recebem o sinal, na forma de ondas

eletromagnéticas que são transmitidas da estação de

televisão. O sinal é exibido em sua tela de televisão.

Companhias de cabo têm antenas que

recebem ondas transmitidas de suas estações de

Page 9: Física Moderna Capítulo 4 - Radiação de corpo negro e ... · de correntes de convecção cde ar aquecido, que se movem para cima. Colocando-se em um dos lados do ... para a densidade

Física Moderna – Capítulo 4 - Radiação de corpo negro e Efeito Fotoelétrico

Prof. Dr. Cláudio S. Sartori

31

31

TELEVISÃO locais. O sinal é enviado então por um

cabo para sua casa.

Os telefones celulares usam ondas de rádio

para transmitir informação. Estas ondas são muito

menores que as da TELEVISÃO e ondas de rádio de

FM.

Por que as antenas estão em telefones

celulares são menores que antenas em seu radio?

Como nós ―vemos‖ usando ondas de rádio?

Objetos no espaço, como planetas e cometas,

nuvens gigantes de gás, estrelas e galáxias, emitem luz

a muitos comprimentos de onda diferentes. Algumas

das luzes que eles emitem tem comprimentos de onda

muito grandes - às vezes quase que um milhão. Estas

ondas longas estão na região de rádio do espectro

eletromagnético.

As ondas de rádio são maiores que ondas

ópticas, e as antenas que captam ondas de rádio

trabalham diferentemente que telescópios que nós

usamos para luz visível (telescópios ópticos). Radio

telescópios são extensas superfícies parabólicas de

metal que refletem ondas de rádio para um ponto

focal. Devido os comprimentos de onda de luz de

rádio serem tão grandes, radiotelescópios devem ser

fisicamente maiores que um telescópio óptico para que

possa fazer imagens de claridade comparável. Por

exemplo, o Parkes rádio telescópio tem uma

circunferência de raio de 64 metros.

Para fazer imagens de rádio melhores (ou

resolução mais alta) o rádio astrônomos combina

freqüentemente vários telescópios menores, ou pratos

receptores, em uma certa ordem. Juntos, os pratos

podem agir como um telescópio grande cujo tamanho

se iguala à área total ocupado pelo conjunto.

O VLA é um dos primeiros observatórios

rádios astronômicos do mundo. O VLA consiste em 27

antenas organizadas em forma de Y ―enorme‖ de até

36 km (22 milhas) uma vez e meias o tamanho de

Washington, DC.

O VLA, localizado em Novo México, é um

interferômetro; isto significa que opera multiplicando

os dados junto de cada par de telescópios para formar

padrões de interferência. A estrutura desses padrões de

interferência, e como eles mudam com o tempo

conforme a Terra gira refletem a estrutura de fontes de

rádio no céu. O que nos mostram as ondas de rádio?

A figura acima mostra espetáculos da imagem

de Monóxido de Carbono (CO) e gases em nossa

galáxia Via Láctea.

Muitos objetos astronômicos emitem ondas

de rádio, fato que não foi descoberto até 1932. Desde

então, astrônomos desenvolveram sistemas

sofisticados que lhes permitem fazer fotografias das

ondas de rádio emitidas por objetos astronômicos. As

ondas de Rádio provenientes dos céus são devido a

planetas e cometas, nuvens gigantes de gás e poeira,

estrelas e galáxias. Estudando as ondas de rádio

originadas destas fontes, astrônomos podem aprender

sobre a composição delas, sua estrutura e movimento.

A Radio Astronomia tem a vantagem que a luz solar,

nuvens, e chuva não afetam as observações.

Radiação infravermelha:

Medidas de luz infravermelha estão

compreendidas entre o visível e as microondas do

espectro eletromagnético. Luz infravermelha tem um

alcance de comprimentos de onda que variam

próximos da luz vermelha a violeta. "Luz

infravermelha próxima" é comparável em

comprimento de onda para luz visível e

"infravermelho longínguo" é próximo à região de

microondas do espectro eletromagnético. Os

comprimentos de onda infravermelhos mais longos

estão próximos ao tamanho de uma cabeça de alfinete

e o infravermelho próximo é microscópico.

Ondas infravermelhas distantes são térmicas.

Em outra palavra, nós experimentamos este tipo de

radiação infravermelha diariamente na forma de

calor! O calor que nós sentimos de luz solar, um

fogo, um radiador ou uma calçada morna é

infravermelha. Os terminais nervosos em nossa pele

possuem sensibilidade para poder descobrir a

diferença entre a temperatura do corpo à temperatura

da pele.

Usamos a radiação infravermelha para aquecer

até mesmo a comida; luminárias especiais que

emitem ondas infravermelhas térmicas são

freqüentemente usadas em restaurantes de comida

rápidas (fast food).

Ondas infra-vermelhas menores, próximas, não

são quentes, de fato você nem mesmo as sente. Estes

comprimentos de onda menores são os usados por

seu o controle remoto de Televisão.

Como nós ―podemos ver‖ usando o

Infravermelho?

Como a fonte primária de radiação

infravermelha é gerada pelo calor ou radiação

térmica, qualquer objeto numa temperatura radia no

infravermelho. Até mesmo objetos que nós pensamos

estarem muito frios, como um cubo de gelo, emite

Page 10: Física Moderna Capítulo 4 - Radiação de corpo negro e ... · de correntes de convecção cde ar aquecido, que se movem para cima. Colocando-se em um dos lados do ... para a densidade

Física Moderna – Capítulo 4 - Radiação de corpo negro e Efeito Fotoelétrico

Prof. Dr. Cláudio S. Sartori

32

32

infravermelho. Quando um objeto não está bastante

quente para radiar luz visível, emitirá a maioria de sua

energia no infravermelho. Por exemplo, carvão quente

pode não emitir luz, mas emite radiação infravermelha

e nós sentimos como calor. Até o mais morno objeto

emite radiação infravermelha.

Humanos, a temperatura de corpo normal, radiam

fortemente no infravermelho a um comprimento de

onda de cerca de 10 mícrons. mm 6101 (Um

mícron é o termo comumente usado em física para um

micrômetro ou um milionésimo de um metro)

Esta imagem (que é cortesia do Processamento

Infra-vermelho e Centro de Análise do CalTech),

ilustra como é a fotografia em infravermelho de um

homem que sustenta uma pinça!

Qual objeto desta imagem você acha ter a

temperatura mais morna?

Como a temperatura dos óculos deste homem se

compara à temperatura da mão dele?

Para fazer fotografias infravermelhas

similares à de cima, podemos usar máquinas

fotográficas especiais e filmes que possuem diferenças

em temperatura, e então colocar diferentes ou falsas

cores a eles. Isto resulta numa fotografia que nossos

olhos podem interpretar.

A imagem (cortesia da Corporação de SE-IR,

Goleta, a CA) é uma fotografia de um gato no infra-

vermelho. As áreas laranja são os mais mornos e as

áreas branco-azuis são os mais frios. Esta imagem nos

dá uma visão diferente de um animal familiar como

também informação que nós não pudéssemos obter de

uma

fotografia visível.

Humanos não podem ver luz infravermelha,

mas você sabia que serpentes, como víboras ou

cascavéis, têm órgão "sensoriais‖ que são usados para

detectar a imagem de luz infravermelha? Isto permite

que a serpente descubra animais de sangue morno, até

mesmo em covas escuras! Algumas serpentes possuem

órgãos sensoriais com até mesmo percepção de

profundidade no infravermelho!

Muitas coisas além de pessoas e animais

emitem luz infravermelha - a Terra, o Sol, objetos

distantes como estrelas e galáxias também o fazem!

Para uma visão da órbita de Terra, se nós estamos

olhando fora em espaço ou descemos em Terra, nós

podemos usar instrumentos a bordo de satélites.

Satélites como o VAI 6 e Landsat 7

observam a Terra com sensores especiais, como esses

a bordo o Landsat 7 satélite, dados de registro sobre a

quantidade de luz infra-vermelha refletida ou emitida

da superfície da Terra.

Outros satélites, como o Satélite de

Astronomia Infra-vermelho (IRAS) observam do

espaço e medem a radiação infravermelha nuvens

grandes de pó e gás que podem formar estrelas,e

galáxias!

O que o infravermelho nos mostra?

Esta é uma imagem infravermelha da Terra

tirada pelo satélite VAI 6 em 1986. Cientistas

observam temperaturas diferentes para determinar

quais partes da imagem são de nuvens, terra e mar.

Baseado nestas diferenças de temperatura, usando

256 cores separadamente, coloriu-se a imagem

tornando uma fotografia realista.

Por que usamos o infravermelho para tirar

uma fotografia da Terra? Enquanto é mais fácil de

distinguir nuvens da Terra no alcance visível, há mais

detalhes no interior das nuvens no infra-vermelho.

Pode-se estudar a estrutura da nuvem.

Por exemplo, nota-se que as nuvens mais

escuras estão mais mornas, enquanto nuvens mais

claras estão mais frescas.

No sudeste do Galápagos, só no oeste da

costa de América do Sul, há um lugar onde você

Page 11: Física Moderna Capítulo 4 - Radiação de corpo negro e ... · de correntes de convecção cde ar aquecido, que se movem para cima. Colocando-se em um dos lados do ... para a densidade

Física Moderna – Capítulo 4 - Radiação de corpo negro e Efeito Fotoelétrico

Prof. Dr. Cláudio S. Sartori

33

33

pode ver capas múltiplas de nuvens distintamente,

com as nuvens mais mornas a mais baixas altitudes,

mais próximo ao oceano que está esquentando.

Nós sabemos, ao olhar umas imagens

infravermelhas de um gato, que muitas coisas emitem

luz infravermelha. Mas muitos objetos também

refletem luz infravermelha, particularmente luz

infravermelha próxima. A radiação infravermelha

próxima não é relacionada à temperatura do objeto que

é fotografado - a menos que o objeto seja muito, muito

quente.

Um filme infravermelho ―fotografa‖ o objeto

porque o Sol (ou alguma outra fonte clara) forneceu

luz infravermelha no filme e é refletido ou absorvido

pelo objeto. Você poderia dizer que refletindo ou

absorvendo infravermelho ajuda a determinar a cor do

objeto - sua cor sendo uma combinação de vermelho,

verde, azule, e infravermelho!

Estudos mostram que a clorofila em plantas

reflete ondas infravermelhas próximas junto com

ondas de luz visíveis. Embora nós não possamos ver as

ondas infravermelhas, eles sempre estão lá.

Outro fenômeno importante relacionada com

a radiação infravermelha é o efeito estufa.

O aumento do gás CO2 pode produzir maior

retenção dessa radiação infravermelha produzida pela

Terra, superaquecendo o Planeta.

Instrumentos a bordo de satélites também

podem tirar fotos de objetos no espaço. A imagem

debaixo da região do centro de nossa galáxia foi tirada

pelo satélite IRAS.

A característica da nebulosa em forma de S,

horizontal que cruza a imagem é o calor emitido pelas

nuvens de poeira do sistema solar.

Radiação Visível:

Ondas claras visíveis são as únicas ondas

eletromagnéticas que nós podemos ver. Nós vemos

estas ondas como as cores do arco-íris. Cada cor tem

um comprimento de onda diferente. Vermelho tem o

comprimento de onda mais longo e violeta tem o

comprimento de onda menor. Quando todas as ondas

são vistas juntas, eles formam a luz branca.

Quando um raio de luz branca passa por um

prisma ou por vapor de água como este arco-íris, a

luz branca separa-se nas cores do espectro claro

visível.

Como nós ―vemos‖ usando Luz Visível?

Os cones em nossos olhos são os receptores

para estas ondas de luz visíveis minúsculas. O Sol é

uma fonte natural para ondas de luz visíveis e nossos

olhos observam a reflexão desta luz solar dos objetos

ao nosso redor.

A cor de um objeto que nós vemos é a cor de

luz refletida. Todas as outras cores são absorvidas.

Lâmpadas incandescentes são outra fonte de

ondas de luz visíveis.

Estas são imagens de Phoenix, Arizona, uma

fotografada por uma nave e outra colorida. Você

pode ver uma diferença entre esta imagem e a

fotografia a seguir?

Page 12: Física Moderna Capítulo 4 - Radiação de corpo negro e ... · de correntes de convecção cde ar aquecido, que se movem para cima. Colocando-se em um dos lados do ... para a densidade

Física Moderna – Capítulo 4 - Radiação de corpo negro e Efeito Fotoelétrico

Prof. Dr. Cláudio S. Sartori

34

34

Há dois tipos de imagens coloridas que

podem ser feitas de dados de satélite – as de cores

verdadeiras e as coloridas artificialmente. Tirar

imagens coloridas, como esta aqui, o satélite que tirou

isto usou um sensor para registrar dados sobre as

ondas de luz visíveis vermelhas, verdes, e azuis que

estavam refletindo a superfície da terra. Os dados

foram combinados num computador mais tarde. O

resultado é semelhante ao que nossos olhos vêem.

Uma imagem de cor falsa é feita quando o

satélite registra dados sobre brilho das ondas claras

que refletem a superfície da Terra. Estes brilhos são

representados por valores numéricos - e estes valores

podem ser codificados por cores. É como pintar

através de números! As cores escolhidas que pintam a

imagem são arbitrárias, mas eles podem ser escolhidos

ou fazer o objeto parecer realista, ou ajudar a enfatizar

uma característica particular na imagem. Astrônomos

podem ver uma região de interesse até mesmo usando

software para mudar o contraste e brilho no quadro,

como os controles em uma TELEVISÃO! Você pode

ver uma diferença nas paletas de cor selecionadas para

as duas imagens abaixo?

Ambas as imagens são da Nebulosa de

Caranguejo, os restos de uma estrela explodida!

O que nos mostra Luz Visível?

É verdade que nós somos cegos a muitos

comprimentos de onda de luz. Por isso usamos

instrumentos que podem descobrir comprimentos de

onda diferentes de luz para nos ajudar a estudar a

Terra e o Universo. Porém, desde que luz visível é

parte do espectro eletromagnético que nossos olhos

podem ver, nosso mundo inteiro é orientado ao redor

disso. E muitos instrumentos que descobrem luz

visível podem ver mais claramente que nossos olhos,

com maior sensibilidade à radiação. Por isso é por que

nós usamos satélites para olhar a Terra, e telescópios

para olhar o Céu!

Nós não só olhamos a Terra do espaço, mas

nós também podemos olhar outros planetas de

espaço. Esta é uma imagem clara visível do planeta

Júpiter. Está em falsa cor - as cores foram escolhidas

para enfatizar a estrutura de nuvem que atua no

planeta Júpiter e não apareceria a seus olhos.

Radiação ultravioleta (UV):

A luz ultravioleta (UV) tem comprimentos

de onda menores que luz visível. Embora estas ondas

sejam invisíveis ao olho humano, alguns insetos,

como abelhas, os podem ver.

Cientistas dividiram a parte ultravioleta do

espectro em três regiões: o ultravioleta próximo, o

ultravioleta distante, e o ultravioleta extremo. As três

regiões são distintas pela energia da radiação

ultravioleta e pelo comprimento de onda da luz

ultravioleta que é relacionada com a energia.

O NUV ultravioleta próximo, abreviado por

NUV – Near Ultra-violet, é a luz mais próxima da luz

óptica ou visível. O ultravioleta extremo, abreviado

EUV, é a luz ultravioleta mais próxima para

Radiografias, e é o mais enérgico dos três tipos. O

ultravioleta distante, abreviado FUV, encontra-se

entre as próximas e extremas regiões ultravioletas. É

o menos explorado das três regiões.

Nosso Sol emite luz a todos os

comprimentos de onda diferentes em espectro

eletromagnético, mas são as ondas ultravioletas que

são responsáveis para causar nossas queimaduras de

sol.

À esquerda é uma imagem do Sol tirada a

um comprimento de onda Ultravioleta Extremo - 171

Angstroms para ser exato. (Um Angstrom é igual a

10-10

metros.) Esta imagem foi tirada por um satélite

denominado SOHO e mostra o Sol em 24 de abril de

2000.

Embora algumas ondas ultravioletas do Sol

penetrem a atmosfera de Terra, a maioria delas são

bloqueadas ao penetrá-la por vários gases como o

Ozônio (O3).

Page 13: Física Moderna Capítulo 4 - Radiação de corpo negro e ... · de correntes de convecção cde ar aquecido, que se movem para cima. Colocando-se em um dos lados do ... para a densidade

Física Moderna – Capítulo 4 - Radiação de corpo negro e Efeito Fotoelétrico

Prof. Dr. Cláudio S. Sartori

35

35

Cientistas desenvolveram um índice de UV

para ajudar as pessoas a se proteger destas ondas

prejudiciais.

Como nós ―vemos‖ usando luz Ultravioleta?

É bom para nós que somos humanos estar

protegido de adquirir muita radiação ultravioleta, mas

é ruim para os cientistas! Astrônomos têm que colocar

telescópios ultravioleta em satélites e medir a luz

ultravioleta de estrelas e galáxias - e coisas até mais

próximas como o Sol!

Há muitos satélites diferentes que nos ajudam

e estudam a astronomia ultravioleta. Muitos deles só

descobrem uma pequena quantidade de luz UV. Por

exemplo, o telescópio espacial Hubble observa estrelas

e galáxias principalmente em luz ultravioleta próxima.

O satélite Explorador Ultravioleta Extremo da NASA

está explorando o universo ultravioleta extremo

atualmente. O satélite Explorador Ultravioleta

Internacional (IUE) observou as regiões ultravioletas

distantes e próximas durante mais de 17 anos.

O que nos mostra luz Ultravioleta?

Nós podemos estudar estrelas e galáxias

estudando o UV que elas emitem - mas você sabia que

nós podemos estudar até mesmo a Terra?

O UV Camera/Spectrograph Distante tirou a

foto a seguir. A parte da Terra iluminada pelo Sol

reflete muita luz UV.

Aqui, faixas de emissão de UV também são

aparentes. Estas faixas são o resultado de aurora

causada por partículas carregadas emitidas pelo Sol.

Elas espiralam para a Terra ao longo das linhas de

campo magnético da Terra.

Muitos cientistas estão interessados em

estudar o universo invisível de luz ultravioleta, desde

os mais quentes e os objetos mais ativos no cosmo que

emitam quantias grandes de energia ultravioleta.

A imagem ao lado ilustra três galáxias

diferentes obtidas em luz visível (fundo três imagens)

e luz ultravioleta (fila de cima) tirada pelo Imaging

Telescope Ultravioleta da NASA (UIT) na missão

Astro-2.

As diferenças de como as galáxias aparecem é

devido ao tipo de brilho de estrelas mais luminosas nos

comprimentos de onda ópticos e ultravioletas. As

fotografias dessas galáxias indicam principalmente

nuvens de gás contendo que formarão estrelas

recentes muitas vezes mais volumosas que o sol que

arde fortemente em luz ultravioleta. Em contraste,

fotografias de luz visíveis de galáxias principalmente

a luz amarela e vermelha são de estrelas mais velhas.

Comparando estes tipos de dados, astrônomos podem

aprender sobre a estrutura e evolução de galáxias.

Radiação X (Raios X): Com a diminuição dos comprimentos de

onda, eles aumentam sua energia. Radiografias têm

comprimentos de onda menores e então energia mais

alta que ondas ultravioletas. Nós normalmente

falamos sobre Radiografias em termos da energia em

lugar de comprimento de onda. Isto porque

Radiografias têm comprimentos de onda muito

pequenos. Também é porque luz de Radiografia

tende a agir mais como uma partícula que uma onda.

Detectores de radiografia absorvem fótons de luz de

Raios X - que é muito diferente dos telescópios de

rádio que têm pratos grandes projetados para detectar

ondas de rádio!

Foram observados as primeiras radiografias

e documentadas em 1895 por Wilhelm Conrad

Röentgen, um cientista alemão que a descobriu

acidentalmente quando estava realizando

experiências com tubos de vácuo.

Uma semana depois, ele levou uma

fotografia de Radiografia da mão de sua esposa que

claramente revelou o anel de casamento dela e seus

ossos. A fotografia assombrou o público em geral e

grande interesse científico foi despertado nessa nova

forma de radiação. Röentgen chamou isto de radiação

―X‖ para indicar que era um tipo desconhecido de

radiação. O nome aderiu, embora (em cima das

objeções de Röentgen), muitos dos seus colegas

questionaram os chamando raios de Röentgen. Eles

ainda são ocasionalmente chamado raios de Röentgen

em países de língua alemã.

A atmosfera da Terra é espessa bastante que

virtualmente nenhuma radiação na faixa dos Raios X

pode penetrar do espaço exterior para a superfície da

Terra. Isto é bom para nós, mas também ruim para

astronomia - temos que pôr telescópios e detectores

de Raios X em satélites! Nós não podemos fazer

astronomia de raios X do solo.

Como nós ―vemos‖ usando os Raios X?

Bem, nós não poderíamos ver pelas roupas

de pessoas, não importa o que os anúncios para

óculos de Raio X nos contam! Se nós pudéssemos

ver os Raios X, nós poderíamos ver coisas que ou

emitem Raio X ou detêm a transmissão deles. Nossos

olhos estariam como o filme de Radiografia usado

em hospitais ou os escritórios de dentista. Filme de

radiografia ―vê‖ Raios X.

Page 14: Física Moderna Capítulo 4 - Radiação de corpo negro e ... · de correntes de convecção cde ar aquecido, que se movem para cima. Colocando-se em um dos lados do ... para a densidade

Física Moderna – Capítulo 4 - Radiação de corpo negro e Efeito Fotoelétrico

Prof. Dr. Cláudio S. Sartori

36

36

Quando você adquire uma Radiografia tirada

em um hospital, um filme sensível é posto em um lado

de seu corpo, e são atiradas Radiografias sobre você.

Num consultório dentário, o filme é posto dentro de

sua boca, em um lado de seus dentes, e são atiradas

Radiografias por sua mandíbula. Não dói nada - você

não pode sentir Raio X.

É porque seus ossos e dentes são densos e

absorvem mais raio X que sua pele, produz-se

silhuetas de seus ossos ou dentes que permanecem no

filme de Radiografia enquanto sua pele aparece

transparente. Metais absorvem mais Raios X - você

pode ver o objeto metálico na imagem do dente?

Quando o Sol nos ilumina num certo ângulo,

nossa sombra é projetada sobre o solo.

Semelhantemente, quando os Raios X incidem em nós,

passa por nossa pele, mas permite projetar sombras

sobre nossos ossos e são capturadas através de um

filme.

Abaixo vemos a fotografia de Radiografia de

uma menina. Você pode ver a sombra do objeto que

ela engoliu?

Ao centro vemos a Radiografia da mão da

esposa de Röentgen.

Nós usamos satélites com detectores de

Raios-X para Radiografar imagens em astronomia. Em

astronomia, objetos que emitem Raios-X (por

exemplo, buracos negros) são como uma máquina de

Radiografia do dentista, e o detector no satélite

funciona como o filme de Radiografia. Detectores de

raios-X absorvem Raios individuais (fótons de luz de

Raios-X) e o número de fótons coletados, a energia

dos fótons, ou quão rápidos os fótons são absorvidos,

podem nos contar informações importantes sobre o

objeto que os está emitindo. À direita há uma imagem

de um detector de Raio X. Este

instrumento está no satélite Explorador (RXTE).

Parece muito diferente de qualquer coisa que você

poderia ver no escritório de um dentista!

O que os Raios X nos mostra?

Muitas coisas no espaço emitem Raios-X,

entre eles estão buracos negros, estrelas de nêutrons,

sistemas binários de estrelas, sobras de supernova,

estrelas, o Sol, e até mesmo alguns cometas!

A Terra emite muitos tipos de luz, inclusive

a faixa de Raios-X enérgica. De fato, a própria Terra

emite - a aurora produz na atmosfera da Terra. Esta

aurora é causada pela incidência na atmosfera de

partículas carregadas do Sol.

A foto é do satélite Polar, PIXIE, NASA e à

esquerda está a primeira fotografia da Terra em

Raios-X, tirada em março de 1996 com o satélite

Polar orbital. A área de emissão de Radiografia mais

luminosa é vermelha. As partículas carregadas

enérgicas do Sol que causam a aurora também

energizam elétrons na magnetosfera da Terra. Estes

elétrons movem sobre o campo magnético da Terra e

eventualmente golpeiam as moléculas da ionosfera da

Terra e causam a emissão de Raios X. Estes Raios X

não são perigosos porque eles são absorvidos por

partes mais baixas da atmosfera da Terra.

Recentemente, nós aprendemos que cometas

emitem Radio X! A imagem acima do Cometa

Hyakutake foi tirada por um satélite de Raios-X

chamado ROSAT.

O Sol também emite Raios X – a foto da

direita é do Sol observado por Radiografia de 27 de

abril de 2000. Esta imagem foi tirada pelo satélite

Yokoh.

Muitas estrelas formam sistemas binários –

duas estrelas que orbitam uma em relação à outra.

Quando uma destas estrelas é um buraco negro ou

uma estrela de nêutrons, material é puxado da estrela

normal. Estas espirais de materiais no buraco negro

ou na estrela de nêutron possuem temperaturas muito

altas. Quando algo é aquecido a mais de um milhão

de graus, emitirá Raios X!

A imagem esquerda anterior é a concepção

de um artista de um sistema de estrela binário e

mostra o material sendo puxado da estrela vermelha

por seu companheiro, um buraco negro invisível e em

um disco de órbita. A imagem à direita mostra uma

sobra de supernova - a sobra de uma estrela que

explodiu em uma galáxia perto conhecida como a

Nuvem de Magalhães Pequena. As colorizações

Page 15: Física Moderna Capítulo 4 - Radiação de corpo negro e ... · de correntes de convecção cde ar aquecido, que se movem para cima. Colocando-se em um dos lados do ... para a densidade

Física Moderna – Capítulo 4 - Radiação de corpo negro e Efeito Fotoelétrico

Prof. Dr. Cláudio S. Sartori

37

37

utilizadas mostram que esta sobra de supernova

emitem em Raio X (em azul), luz visível (verde) e de

rádio (vermelho).

Radiação Gama (Raios ):

Os Raios Gama têm os comprimentos de onda

menores e a maior energia de qualquer outra onda no

espectro eletromagnético. Estas ondas são geradas

através de átomos radioativos e em explosões

nucleares.

Raios Gama podem matar células vivas, um

fato que a medicina utiliza para matar células

cancerosas.

Os raios Gama viajam a nós por distâncias

vastas do universo e são absorvido pela atmosfera da

Terra. Comprimentos de onda diferentes de luz

penetram a atmosfera da Terra para profundidades

diferentes. Instrumentos a bordo balões de alta-altitude

e satélites como o Observatório de Compton provêem

nossa única visão do céu de gama-raio.

Raios Gama são a forma mais enérgica de luz

e são produzidos pelas regiões mais quentes do

universo. Eles também são produzidos através de

eventos violentos como explosões de supernova ou a

destruição de átomos, e através de eventos como o

decaimento de material radioativo no espaço. Coisas

como explosões de supernova (o modo como as

estrelas volumosas morrem), estrelas de nêutrons,

pulsares e buracos negros são todas as fontes celestiais

de raios gama.

Como nós ―vemos‖ usando luz de raios-gama?

A Astronomia de raio-gama não se desenvolveu

até que fosse possível colocar detectores acima da

atmosfera, usando balões ou satélites. O primeiro

telescópio de raios gama, levado em órbita pelo

satélite Explorador XI em 1961, capturou menos que

100 fótons de raios gama cósmicos!

Luz óptica distinta e Radiografias não podem ser

utilizadas para capturar raios gama e podem ser

refletidos em espelhos. Os fótons de alta-energia

passariam direto por tal dispositivo. Para detectar raios

Gama usa-se um processo chamado Compton, onde

um fóton de raio gama golpeia um elétron e perde

energia, semelhante a uma bola que golpeia uma outra

bola.

O que nos mostram os raios gama?

Se você pudesse ver a radiação gama, o céu

noturno pareceria estranho e pouco conhecido.

A lua de vista por meio de raios gama emitida

por ela apareceria como uma redonda gota -

características lunares não seriam visíveis. Em raios de

gama de alto-energia, a Lua é realmente mais luminosa

que o Sol. As visões familiares de estrelas e galáxias

seriam substituídas por algo sempre variável. Sua

visão de raio gama apareceria labaredas solares,

supernovas, estrelas de nêutons, buracos negros, e

galáxias ativas. Astronomia de raios gama

apresentam oportunidades sem igual para explorar

estes objetos exóticos. Explorando o universo a estas

altas energias, cientistas podem procurar uma nova

física, testar teorias e executar experiências que não

são possíveis em laboratórios da Terra.

Se você pudesse ver raios raios gama,

estrelas de nêutrons ou pulsares estariam entre os

objetos mais luminosos no céu. Este computador

processou imagem que mostra o pulsar de Nebulosa

de Caranguejo (debaixo de e a direito do centro) e o

pulsar de Geminga (sobre e a partir do

centro) na "faixa da luz" de raios gama raios.

A nebulosa de Caranguejo, também

mostrada na imagem clara visível, foi criada por uma

supernova que clareou o céu noturno em 1054 D.C.

Em 1967, astrônomos descobriram o caroço que

sobrou daquela estrela; um rápido pulsar giratório,

magnético que produz ondas de rádio a cada 0.33

segundos.

Talvez a descoberta mais espetacular em

astronomia de raios gama ocorreu nos anos 1960s e

setenta. Um Detector a bordo do satélite Vela,

satélites originalmente militares, começou a registrar

estouros de gama-raios não da Terra, mas do espaço

profundo!

Explosões de raio gama podem lançar mais

energia em 10 segundos que o Sol emitirá em sua

vida inteira de 10 bilhões de anos! Tão longe, aparece

que os estouros que nós observamos vieram de fora

da Galáxia da Via Láctea. Os cientistas acreditam

que as explosões de raio gama acontecem a alguns

milhões de anos na Via Láctea, e de fato pode

acontecer uma vez a cada cem milhões de anos e

dentro de alguns mil ano-luz da Terra.

Estudado agora durante mais de 25 anos

com instrumentos a bordo de uma variedade de

satélites e sondas de espaço, inclusive astronave de

Venera soviética e o Vênus Orbiter Pioneiro, as

fontes destes flash de alto-energia enigmáticos

permanecem um mistério.

Page 16: Física Moderna Capítulo 4 - Radiação de corpo negro e ... · de correntes de convecção cde ar aquecido, que se movem para cima. Colocando-se em um dos lados do ... para a densidade

Física Moderna – Capítulo 4 - Radiação de corpo negro e Efeito Fotoelétrico

Prof. Dr. Cláudio S. Sartori

38

38

Resolvendo o mistério de estouros de gama-

raio, cientistas esperam ganhar conhecimento

adicional das origens do Universo, a taxa à qual o

Universo está se expandindo, e o tamanho do

Universo.

Microondas

As Microondas têm comprimentos de onda

que podem ser medidos em centímetros! A microonda

mais longa, esses mais próximo de um pé, são as

ondas que aquecem nossa comida em um forno de

microondas.

Microondas são bons para transmitir

informação de um lugar para outro porque energia da

microonda pode penetrar névoa, chuva clara e neva,

nuvens, e fumaça.

Microondas menores são usados sentindo

distante. Esta microonda é usada para radar como o

radar Doppler usado em previsões de tempo.

A torre de microondas pode transmitir

informações para telefonia celular e dados de

computador de uma cidade para outra.

Os radares foram desenvolvidos para

descobrir objetos e determinar o alcance deles (ou

posição) transmitindo pequenos sinais de microondas.

São registradas a força e origem de "ecos" recebidas

de objetos que foram rebatidos pelas microondas.

O radar detecta ondas eletromagnéticas que

são uma reflexão de uma transmissão ativa, e é

considerado um sistema distante ativo. Sistema

distante passivo se refere ao sentindo de ondas

eletromagnéticas que não originaram do satélite ou

sensor. O sensor é um observador passivo.

Bandas de Frequências para o Celular

no Brasil Estão disponíveis para o celular no Brasil

(SMP) frequências nas bandas de:

850 MHz, antigas bandas A e B

900 MHz, bandas de extensão utilizadas pelo

GSM.

1700 e 1800 MHz, bandas D, E e subfaixas de

extensão utilizadas pelo GSM

1900 e 2100 MHZ destinadas na sua maior

parte para sistemas 3G.

Freqüências

(MHz)

Transmissão da

Estação

Móvel ERB

Subfaixa

A**

824-835

845-846,5

869-880

890-891,5

Subfaixa

B**

835-845

846,5-849

880-890

891,5-894

Subfaixa D 910-912,5 1710-1725

955-957,5 1805-1820

Subfaixa E 912,5-915

1740-1755

957,5-960

1835-1850

Subfaixas

de Extensão

898,5-901*

907,5-910*

1725-1740

1775-1785

943,5-946*

952,5-955*

1820-1835

1870-1880

* Não serão autorizadas para prestadoras do SMP operando nas Bandas D e E. Todas as operadoras de Banda D e E adquiriram

também as faixas de frequências de 900 MHz alocadas para a sua

Banda. ** Admite o emprego de sistemas analógicos (AMPS) nas Bandas

A e B até 30/06/2008.

Novas Bandas do SMP

Res. 454 de 11/12/06 que revogou a Res. 376 02/09/04.

MHz Transmissão da

Subfaixa Estação

Móvel ERB

F* 1920-1935 2.110-2.125

G* 1.935-1.945 2.125-

2.135

H* 1.945-1.955 2.135-

2.145

I* 1.955-1.965 2.145-2.155

J* 1.965-1.975 2.155-

2.165

L 1.895-1.900 1.975-

1.980

M 1.755-1.765 1.850-1.860

Subfaixa de

Extensão

1.765-1.770 1.770-1.775

1.860-

1.865 1.865-

1.870

1.885-1.890** 1.890-1.895**

* Faixas reservadas para sistemas 3G ** Sistemas TDD (Time Division Duplex) que utilizam a mesma

subfaixa de frequências para transmissão nas duas direções.

Page 17: Física Moderna Capítulo 4 - Radiação de corpo negro e ... · de correntes de convecção cde ar aquecido, que se movem para cima. Colocando-se em um dos lados do ... para a densidade

Física Moderna – Capítulo 4 - Radiação de corpo negro e Efeito Fotoelétrico

Prof. Dr. Cláudio S. Sartori

39

39

http://www.teleco.com.br/Bandac.asp

Como as microondas podem penetrar névoa,

chuva clara e neva, nuvens e fuma, estas ondas são

boas para ver a Terra do espaço.

O ERS-1 satélite envia comprimentos de onda

aproximadamente 5.7 cm (faixa-C).

O satélite de JERS usa comprimentos de

onda aproximadamente 20 cm em duração (faixa L).

Nos anos de 1960 uma descoberta

surpreendente foi feita totalmente através de acidente.

Um par de cientistas em Laboratórios de Sino

descobriu ruído de fundo utilizando uma antena de

rádio especial. A coisa estranha sobre o ruído era que

estava vindo de toda direção e não parecia variar em

intensidade muito nada. Se esta fosse estática de algo

em nosso mundo, iguais transmissões de rádio de um

aeroporto perto que controlava a torre, só viria de uma

direção, não em todos lugares. Os cientistas

perceberam logo que eles tinham descoberto a

radiação de fundo de microonda cósmica. É acreditado

que esta radiação que enche o Universo inteiro é

devida ao conhecido Big Bang.

Page 18: Física Moderna Capítulo 4 - Radiação de corpo negro e ... · de correntes de convecção cde ar aquecido, que se movem para cima. Colocando-se em um dos lados do ... para a densidade

Física Moderna – Capítulo 4 - Radiação de corpo negro e Efeito Fotoelétrico

Prof. Dr. Cláudio S. Sartori

40

40

Page 19: Física Moderna Capítulo 4 - Radiação de corpo negro e ... · de correntes de convecção cde ar aquecido, que se movem para cima. Colocando-se em um dos lados do ... para a densidade

Física Moderna – Capítulo 4 - Radiação de corpo negro e Efeito Fotoelétrico

Prof. Dr. Cláudio S. Sartori

41

41

Apêndice

Espectros de estrelas (Adaptado de:

http://docs.kde.org/stable/pt_BR/kdeedu/kstars/ai-colorandtemp.html)

As estrelas parecem ser exclusivamente

brancas a primeira vista. Mas se olharmos

cuidadosamente, podemos notar uma faixa de cores:

azul, branco, vermelho e até dourado. Na constelação

de Orion, um bonito contraste é visto entre o vermelho

de Betelgeuse no "sovaco" de Orion e o azul de

Bellatrix no ombro. O que faz estrelas exibirem cores

diferentes permanecia um mistério até dois séculos

atrás, quando físicos obtiveram suficiente

conhecimento da natureza da luz e propriedades da

matéria em temperaturas imensamente altas.

Especificamente, foi a física da radiação dos

corpos negros que nos possibilitou entender a variação

das cores estelares. Logo após o entendimento do que

era a radiação dos corpos negros, notou-se que o

espectro das estrelas parecia extremamente similar as

curvas da radiação dos corpos negros em várias

temperaturas, variando de poucos milhares de Kelvin

até 50.000 Kelvin. A conclusão óbvia é que estrelas

são semelhantes a corpos negros, e que a variação de

cor das estrelas é uma consequência direta da

temperatura de sua superfície.

Estrelas frias (isto é, Espectro Tipo K e M)

irradiam a maior parte de sua energia na região

vermelha e infravermelha do espectro

electromagnético e assim parecem vermelhas,

enquanto estrelas quentes (isto é, Espectro Tipo O e B)

emitem principalmente em comprimentos de onda azul

e ultravioleta, fazendo-as parecerem azul ou brancas.

Para estimar a temperatura superficial de uma

estrela, podemos usar a conhecida relação entre

temperatura de um corpo negro e o comprimento de

onda da luz no pico de seu espectro. Isto é, conforme

você aumenta a temperatura de um corpo negro, o pico

de seu espectro move-se para um menor (mais azul)

comprimento de onda luminoso. Isto é ilustrado na

Figura 1 abaixo onde a intensidade de três estrelas

hipotéticas é plotada contra o comprimento de onda. O

"arco-íris" indica a faixa de comprimento de onda que

é visível ao olho humano.

Figura 1 – Espectro de estrelas de diferentes cores.

Este método simples é conceitualmente

correto, mas não pode ser usado para obter

temperaturas estelares precisas, porque

estrelas não são corpos negros perfeitos. A presença

de vários elementos na atmosfera estelar fará com

que alguns comprimentos de onda sejam absorvidos.

Devido a estas linhas de absorção não serem

uniformemente distribuídas no espectro, elas podem

inclinar a posição do pico espectral. Além disso,

obter um espectro estelar é um processo de tempo

intensivo e é proibitivamente difícil para grandes

amostras de estrelas.

Um método alternativo utiliza a fotometria

para medir a intensidade da luz passando por

diferentes filtros. Cada filtro permite apenas uma

parte específica do espectro passar enquanto todas as

outras são rejeitadas. Um sistema fotométrico muito

utilizado chama-se sistema UBV Johnson. Ele

emprega três filtros de banda: U ("Ultra-violeta"), B

("Azul"), and V ("Visível"), cada uma ocupando as

diferentes regiões do espectro eletromagnético.

O processo de fotometria UBV envolve usar

dispositivos foto sensíveis (como filmes ou câmeras

CCD) e mirar um telescópio em uma estrela para

medir a intensidade da luz que passa por cada filtro

individualmente. Este processo fornece três

luminosidades aparentes ou fluxos (quantidade de

energia por cm2 por segundo) designados por Fu, Fb

e FV. A relação dos fluxos Fu/Fb e Fb/Fv é uma

medida quantitativa da "cor" da estrela, e estas

relações podem ser usadas para estabelecer uma

escala de temperatura para estrelas. Falando

genericamente, quanto maiores as relações Fu/Fb e

Fb/Fv de uma estrela, mais quente é sua temperatura

de superfície.

Por exemplo, a estrela Bellatrix em Orion

tem um Fb/Fv = 1,22, indicando que é mais brilhante

pelo filtro B que pelo filtro V. Além disso, sua razão

Fu/Fb é 2,22, então é mais brilhante pelo filtro U.

Isto indica que a estrela deve ser muito quente

mesmo, pois seu pico espectral deve estar em algum

lugar na faixa do filtro U, ou até mesmo em

comprimentos de onda mais baixos. A temperatura

superficial de Bellatrix (determinada por comparação

de seu espectro com modelos detalhados que

conferem com suas linhas de absorção) é perto de

25.000 Kelvin.

Podemos repetir esta análise para a estrela

Betelgeuse. Suas razões Fb/Fv e Fu/Fb são 0.15 e

0.18 respectivamente, então ela é mais brilhante em

V e mais opaca em U. Então, o pico espectral de

Betelgeuse deve estar em algum lugar na faixa do

filtro V, ou mesmo em um comprimento de onda

superior. A temperatura superficial de Betelgeuse é

de apenas 2,400 Kelvin.

Os astrônomos preferem expressar as cores

estelares em termos de diferença em magnitudes, do

que uma razão de fluxos. Assim, voltando para a azul

Bellatrix temos um índice de cor igual a

B - V = -2.5 log (Fb/Fv) = -2.5 log (1.22) = -0.22,

Page 20: Física Moderna Capítulo 4 - Radiação de corpo negro e ... · de correntes de convecção cde ar aquecido, que se movem para cima. Colocando-se em um dos lados do ... para a densidade

Física Moderna – Capítulo 4 - Radiação de corpo negro e Efeito Fotoelétrico

Prof. Dr. Cláudio S. Sartori

42

42

Similarmente, o índice de cor para a vermelha

Betelgeuse é

B - V = -2.5 log (Fb/Fv) = -2.5 log (0.18) = 1.85

Os índices de cores, como a escala de

magnitude ,correm para trás. Estrelas Quentes e

azuis têm valores de B-V menores e negativos que as

mais frias e vermelhas estrelas.

Um Astrônomo pode então usar os índices de

cores para uma estrela, após corrigir o

avermelhamento e extinção interestelar, para obter

uma precisa temperatura daquela estrela. A relação

entre B-V e temperatura é ilustrada na Figura 2.

Figura 2 – Relação B-V e temperatura.

Pirômetros Um pirómetro (também denominado de

pirómetro óptico) é um dispositivo que mede

temperatura sem contacto com o corpo/meio do qual

se pretende conhecer a temperatura. Geralmente este

termo é aplicado a instrumentos que medem

temperaturas superiores a 600 graus celsius. Uma

utilização típica é a medição da temperatura de metais

incandescentes em fundições.

Um dos pirómetros mais comuns é o de

absorção-emissão, que é utilizado para determinar a

temperatura de gases através da medição da radiação

emitida por uma fonte de referência, antes e depois da

radiação incidir sobre o gás (que absorve parte da

radiação). É através da análise das diferenças do

espectro do gás que se consegue determinar a sua

temperatura. Ambas as medições são feitas no mesmo

intervalo de comprimento de onda.

Outra aplicação típica do pirómetro é a

medição da temperatura de metais incandescentes.

Olhando pelo visor do pirómetro observa-se o metal,

ajustando-se depois manualmente a corrente elétrica

que percorre um filamento que está no interior do

pirómetro e aparece no visor. Quando a cor do

filamento é idêntica à do metal, pode-se ler a

temperatura numa escala disposta junto ao elemento de

ajuste da cor do filamento.

Page 21: Física Moderna Capítulo 4 - Radiação de corpo negro e ... · de correntes de convecção cde ar aquecido, que se movem para cima. Colocando-se em um dos lados do ... para a densidade

Física Moderna – Capítulo 4 - Radiação de corpo negro e Efeito Fotoelétrico

Prof. Dr. Cláudio S. Sartori

43

43

Descoberto por acaso o sucessor das

lâmpadas incandescentes Redação do Site Inovação Tecnológica

25/10/2005 http://www.inovacaotecnologica.com.br/

Pegue um LED que produza uma luz azul

intensa. Recubra-o com uma finíssima película de

cristais microscópicos, chamados pontos quânticos, e

você terá a próxima revolução tecnológica na

iluminação, que poderá substituir virtualmente todas as

atuais lâmpadas.

Esse LED híbrido, descoberto por acaso pelo

estudante Michael Bowers, da Universidade

Vanderbilt, Estados Unidos, é capaz de emitir luz

branca verdadeira, similar à emitida pelas lâmpadas

incandescentes, com uma leve tonalidade de amarelo.

Até agora os pontos quânticos têm recebido

atenção graças à sua capacidade de produzir dezenas

de cores diferentes simplesmente variando-se o

tamanho dos nanocristais individuais: uma capacidade

particularmente adequada à marcação fluorescente de

células em aplicações biomédicas.

Mas os cientistas agora descobriram uma

nova forma para construir pontos quânticos capazes de

produzir espontaneamente luz branca de largo

espectro.

Até 1993 os LEDs só produziam luzes

vermelha, verde e amarela. Foi então que o

pesquisador japonês Isamu Akasaki descobriu como

fabricar LEDs que emitiam luz azul. Combinando

LEDs azuis com outros verdes e vermelhos - ou

adicionando-se fósforo amarelo aos LEDs azuis - os

fabricantes conseguiram criar luz branca, o que abriu

uma gama totalmente nova de aplicações para essas

fontes de luz, por natureza extremamente econômicas

e duráveis. Mas a luz emitda pelos "LEDs brancos" é

apenas ligeiramente branca, apresentando um forte

tom azulado.

Os pontos quânticos de luz branca, por outro

lado, produzem uma distribuição mais suave dos

comprimentos de onda do espectro visível, com uma

leve tonalidade amarela. Desta forma, a luz produzida

pelos pontos quânticos se parece mais com as luzes de

"espectro total" utilizadas para leitura, um tipo de

lâmpada disponível no mercado que produz uma luz

com um espectro mais próximo ao da luz do Sol do

que as lâmpadas incandescentes ou fluorescentes.

Além disso, os pontos quânticos, como

acontece também com os LEDs, têm a vantagem de

não emitir grandes quantidades de luz infravermelha,

como acontece com as lâmpadas incandescentes. Essa

radiação invisível produz grandes quantidades de calor

e é responsável pela baixa eficiência energética desse

tipo de lâmpada.

Bowers estava estudando com seu colega

James McBride, procurando entender como os pontos

quânticos crescem. Para isso eles estavam tentando

criar pontos quânticos cada vez menores. Foi então

que eles criaram um lote desses nanocristais de cádmio

e selênio. Esses elementos contêm 33 ou 34 pares de

átomos, o que é justamente o "tamanho mágico" no

qual o cristais preferencialmente se formam. Assim,

esses minúsculos pontos quânticos são fáceis de

serem produzidos, ainda que tenham apenas metade

do tamanho dos pontos quânticos normais.

Quando esses pontos quânticos foram

iluminados com um laser, ao invés da luz azul que os

estudantes esperavam, eles se encantaram com o

branco vivo que iluminou a mesa onde faziam seu

experimento.

A seguir os estudantes dissolveram seus

pontos quânticos em uma espécie de verniz para

madeira e "pintaram" um LED. Embora isso seja o

que se poderia chamar de uma típica uma idéia de

estudante, eles estavam, na verdade, montando sua

descoberta sobre uma fonte própria de luz,

dispensando o laser. O resultado não é nenhum

primor de acabamento, mas demonstra claramente

que a junção dos dois pode gerar uma nova fonte de

luz branca que poderá revolucionar todo o setor de

iluminação.

A descoberta foi descrita em um artigo

publicado no exemplar de 18 de Outubro do Jornal da

Sociedade Americana de Química.

Page 22: Física Moderna Capítulo 4 - Radiação de corpo negro e ... · de correntes de convecção cde ar aquecido, que se movem para cima. Colocando-se em um dos lados do ... para a densidade

Física Moderna – Capítulo 4 - Radiação de corpo negro e Efeito Fotoelétrico

Prof. Dr. Cláudio S. Sartori

44

44

Page 23: Física Moderna Capítulo 4 - Radiação de corpo negro e ... · de correntes de convecção cde ar aquecido, que se movem para cima. Colocando-se em um dos lados do ... para a densidade

Física Moderna – Capítulo 4 - Radiação de corpo negro e Efeito Fotoelétrico

Prof. Dr. Cláudio S. Sartori

45

45

Exemplos Resolvidos

1. Determine o fluxo de calor através da barra de cobre

de seção quadrada da figura.

Dado: condutividade térmica do cobre:Cu

388 J(s m°C)-1

e

TA

dt

dQ

4

cals

388 4 10 100 0 =77,6

0,2

dQ A T

dt e

2. Tira-se de uma fornalha uma peça fundida

pesando 50 kgf, quando a temperatura era de 400°C,

sendo colocada num tanque contendo 400 kg de óleo a

30°C. A temperatura final é de 40°C e o calor

específico do óleo, 0,5 cal-g-1

(0C)

-1. Qual o calor

específico da peça fundida? Desprezar a capacidade

calorífica do tanque e quaisquer perdas de calor.

0 0o p o o o p p pQ Q m c m c

400 0,5 40 30 50 40 400 0pc

00,11 calp g C

c

3. A evaporação do suor é um mecanismo

importante no controle da temperatura em animais de

sangue quente. Que massa de água deverá evaporar-se

da superfície de um corpo humano de 80 kg para

resfriá-lo 1°C? O calor específico do corpo humano é

aproximadamente l cal g -1

• (°C) -1

e o calor latente de

vaporização da água na temperatura do corpo (37°C) é

de 577 cal • g -1

.

Quantidade de calor perdida pelo corpo

humano na variação de 10C:

80000 1 1 80000Q mc Q cal

80000138.65

577

LL v

v

QQ mL m g

L

4. Uma certa massa de gás tem o volume de

2,5 L a 40°C na pressão de 1,5 atm. Se o volume do

gás for reduzido para 0.5L e aquecido a 70°C, qual sua

nova pressão?

1 1 2 2 2 1

2 1

1 2 1 2

343 2,51,5 8.22

313 0,5

P V P V T VP P atm

T T T V

5. Para as radiações abaixo, dados os

intervalos extremos de comprimento de onda,

encontre os intervalos correspondentes em freqüência

(Hz) e energia (eV).

Espectro

visível

Visible

Cores maxmin

(nm)

maxminf f f

(1014 Hz)

cf

maxminE E E

(eV)

1240E eV

nm

Red – Vermelho

622 -770 3,896 – 4,823

1,61 – 1,99

Orange – Laranja

597 - 622 4,823 – 5,025

1,99 – 2,08

Yellow –

Amarelo

577 - 597

Green –

Verde

492 - 577

Blue – Azul 455 - 492

Violet –

Violeta

390 - 455

cc f f

814

max9

3 104,823 10

622 10f f

814

min9

3 103,8961 10

770 10f f

E h f

346,62 10h J s

cE h

834 3 10

6,62 10E

251,986 10E J

m

1eV=1,6 10-19

J

25

19 9

1 1,986 10

1,6 10 10E eV

nm

1240E eV

nm

min min

12401,61

770E eV E eV

min max

12401,99

622E eV E eV

Page 24: Física Moderna Capítulo 4 - Radiação de corpo negro e ... · de correntes de convecção cde ar aquecido, que se movem para cima. Colocando-se em um dos lados do ... para a densidade

Física Moderna – Capítulo 4 - Radiação de corpo negro e Efeito Fotoelétrico

Prof. Dr. Cláudio S. Sartori

46

46

Exercícios

1. Condução através de uma geladeira de

isopor. Uma caixa de isopor usada para manter

bebidas frias em um piquenique possui área total

(incluindo a tampa) igual a 0.80m2 e a espessura da

parede é de 2.0 cm. Ela está cheia de água, gelo e latas

de Omni-Cola a 00C. Qual é a taxa de fluxo de calor

para o interior da caixa se a temperatura da parede

externa for igual a 300C? Qual a quantidade de gelo

que se liquefaz durante um dia?

Dado: isopor=0.010 W/(m.K)

2. Uma barra de aço de 10.0 cm de comprimento é

soldada pela extremidade com uma barra de cobre de

20.0 cm de comprimento. As duas barras são

perfeitamente isoladas em suas partes laterais. A seção

reta das duas barras é um quadrado de lado 2.0 cm. A

extremidade livre da barra de aço é mantida a 1000C

colocando-a em contato com vapor d’água obtido por

ebulição, e a extremidade livre da barra de cobre é

mantida a 00C colocando-a em contato com o gelo.

Calcule a temperatura na junção entre as duas barras e

a taxa total da transferência de calor.

e

TA

dt

dQ

eR

A

3. No exemplo anterior, suponha que as barras

estejam separadas. Uma extremidade é mantida a

1000C e a outra extremidade é mantida a 0

0C. Qual a

taxa total de transferência de calor nessas duas barras?

4. Radiação do corpo humano. Sabe-se que a

área total do corpo humano é igual a 1.20m2 e que a

temperatura da superfície é 300C = 303K. Calcule a

taxa total de transferência de calor do corpo por

radiação. Se o meio ambiente está a uma temperatura

de 200C, qual é a taxa resultante do calor perdido

pelo corpo por radiação? A emissividade e do corpo é

próxima da unidade, independentemente da cor da

pele.

Dados: Lei de Stefan-Boltzmann: 4H A e T

4 4

sH A e T T

Constante de Stefan-Boltzmann:

8

2 45.67 10

W

m K

5. Uma placa quadrada de aço, com lado igual a

10 cm, é aquecida em uma forja de ferreiro até uma

temperatura de 8000C. Sabendo que a emissividade é

igual a 0.60, qual é a taxa total de energia transmitida

por radiação?

6. Um chip com embalagem de cerâmica de 40

pinos possui rtérm = 40 K/W. Se a temperatura

máxima que o circuito pode tolerar com segurança

não pode superar 1200C, qual é o mais elevado nível

de potência que o circuito pode tolerar com

segurança para uma temperatura ambiente igual a

750C?

7. Tira-se de uma fornalha uma peça fundida

pesando 50 kgf, quando a temperatura era de 400°C,

sendo colocada num tanque contendo 400 kg de óleo

a 30°C. A temperatura final é de 40°C e o calor

específico do óleo, 0,5 cal-g-1

(0C)

-1. Qual o calor

específico da peça fundida? Desprezar a capacidade

calorífica do tanque e quaisquer perdas de calor.

0 0o p o o o p p pQ Q m c m c

400 0,5 40 30 50 40 400 0pc

00,11 calp g C

c

8. A evaporação do suor é um mecanismo

importante no controle da temperatura em animais de

sangue quente. Que massa de água deverá evaporar-

se da superfície de um corpo humano de 80 kg para

resfriá-lo 1°C? O calor específico do corpo humano é

Page 25: Física Moderna Capítulo 4 - Radiação de corpo negro e ... · de correntes de convecção cde ar aquecido, que se movem para cima. Colocando-se em um dos lados do ... para a densidade

Física Moderna – Capítulo 4 - Radiação de corpo negro e Efeito Fotoelétrico

Prof. Dr. Cláudio S. Sartori

47

47

aproximadamente l cal g -1

• (°C) -1

e o calor latente de

vaporização da água na temperatura do corpo (37°C) é

de 577 cal • g -1

.

Quantidade de calor perdida pelo corpo

humano na variação de 10C:

80000 1 1 80000Q mc Q cal

80000138.65

577

LL v

v

QQ mL m g

L

9. Para as radiações abaixo, dados os

intervalos extremos de comprimento de onda, encontre

os intervalos correspondentes em freqüência (Hz) e

energia (eV).

Espectro visível

Visible

Cores maxmin

(nm)

maxminf f f

(1014

Hz)

cf

maxminE E E

(eV)

1240E eV

nm

Red – Vermelh

o

622 -770 3,896 –

4,823

1,61 –

1,99

Orange –

Laranja 597 - 622 4,823 –

5,025

1,99 –

2,08

Yellow –

Amarelo 577 - 597

Green –

Verde 492 - 577

Blue –

Azul 455 - 492

Violet – Violeta

390 - 455

cc f f

814

max9

3 104,823 10

622 10f f

814

min9

3 103,8961 10

770 10f f

E h f

346,62 10h J s

cE h

834 3 10

6,62 10E

251,986 10E J

m

1eV=1,6 10-19

J

25

19 9

1 1,986 10

1,6 10 10E eV

nm

1240E eV

nm

min min

12401,61

770E eV E eV

10. Área do filamento de uma lâmpada de

tungstênio. A temperatura de operação do filamento

de tungstênio de uma lâmpada incandescente é igual

a 2450K e sua emissividade é igual a 0.35. Calcule a

área da superfície do filamento de uma lâmpada de

150 W supondo que toda a energia elétrica

consumida pela lâmpada seja convertida em ondas

eletromagnéticas pelo filamento. (Somente uma

fração do espectro irradiado corresponde à luz

visível.)

11. Raios de estrelas. A superfície quente e

brilhante de uma estrela emite energia sob a forma de

radiação eletromagnética. É uma boa aproximação

considerar e = 1 para estas superfícies. Calcule os

raios das seguintes estrelas (supondo que elas sejam

esféricas):

(a) Rigel, a estrela brilhante azul da

constelação Órion, que irradia energia com uma taxa

de 2.7.1032

W e a temperatura na superfície é igual a

11000K.

(b) Procyon B (somente visível usando um

telescópio), que irradia energia com uma taxa de

2.1.1023

W e a temperatura na sua superfície é igual a

10000K.

(c) Compare suas respostas com o raio da

Terra, o raio do Sol e com a distância entre a Terra e

o Sol. (Rigel é um exemplo de uma estrela

supergigante e Procyon B é uma estrela anã branca.

12. Determine o comprimento da barra

indicado para que o fluxo de calor seja de 250W.

Dados: condutividade térmica:

cobre:Cu 385,0 J(s m°C)

-1

aço:Aço 50,2 J(s m°C)

-1

Page 26: Física Moderna Capítulo 4 - Radiação de corpo negro e ... · de correntes de convecção cde ar aquecido, que se movem para cima. Colocando-se em um dos lados do ... para a densidade

Física Moderna – Capítulo 4 - Radiação de corpo negro e Efeito Fotoelétrico

Prof. Dr. Cláudio S. Sartori

48

48

13. A Lei do deslocamento de Wien é

obtida, impondo-se 0T

Para:

d

e

hcd

Tk

chT

1

185

Utilizando a Lei do deslocamento de Wien:

max

2.898mm K

T

Ache a que temperatura corresponde ao máximo

comprimento de max = 305 nm.

(b) Aplicando a Lei de Stefan-Boltzman: 4H A e T

: constante de Stefan-Boltzmann. 2

4

85.6699 10 W m

K

Encontre a potência dissipada nessa temperatura,

assumindo área 20 cm2 e emissividade e = 1;

14. Duas barras metálicas, cada qual com 5

cm de comprimento e seção reta retangular de 2 cm

por 3 cm, estão montadas entre duas paredes, uma

mantida a 100 0C e outra a 0

0C. Uma barra é de

chumbo (Pb) e a outra é de prata (Ag). Calcular:

(a) A corrente térmica através das barras.

(b) a temperatura da superfície de contato das

duas.

Dado: Condutividades térmicas:

Pb = 353 W/(m.K)

Ag = 429 W/(m.K)

15. As duas barras do exemplo anterior são

montadas como ilustra a figura a seguir. Calcular:

(a) A corrente térmica em cada barra

metálica.

(b) A corrente térmica total.

(c) A resistência térmica equivalente desta

montagem.

16. A temperatura superficial do Sol é cerca

de 6000K.

(a) Se admitirmos que o Sol irradia como

um corpo negro, em que comprimento de onda max

se localizará o máximo da distribuição espectral?

(b) Calcular max para um corpo negro a

temperatura ambiente, cerca de 300 K.

17. Calcular a perda de energia líquida de

uma pessoa nua numa sala a 200C, admitindo que

irradie como um corpo negro de área superficial igual

a 1.4 m2, na temperatura de 33

0C. A temperatura

superficial do corpo humano é ligeiramente mais

baixa que a temperatura interna de 370C, em virtude

da resistência térmica da pele.

18. Na prática de construção civil, nos países

de língua inglesa, especialmente nos Estados Unidos,

é costume utilizar o fator R, simbolizado por Rf, que é

a resistência térmica por pé quadrado do material.

Assim, o fator R é igual ao quociente entre a

espessura do material e a condutividade térmica:

f

eR R A

A tabela ilustra os fatores de R para alguns

materiais de construção.

Tabela 1 – Fatores R para alguns materiais

de construção.

Material

e

(in)

Rf

(h.ft2.F/Btu)

Chapas divisórias

Gesso ou estuque

0.375 0.32

Compensado

(pinho)

0.5 0.62

Painéis de madeira 0.75 0.93

Carpetes 1.0 2.08

Isolamento de teto 1.0 2.8

Manta asfáltica 1.0 0.15

Chapas de madeira

asfáltica

1.0 0.44

Page 27: Física Moderna Capítulo 4 - Radiação de corpo negro e ... · de correntes de convecção cde ar aquecido, que se movem para cima. Colocando-se em um dos lados do ... para a densidade

Física Moderna – Capítulo 4 - Radiação de corpo negro e Efeito Fotoelétrico

Prof. Dr. Cláudio S. Sartori

49

49

Um telhado de 60 ft por 20 ft é feito de chapa

de pinho, de 1 in, cobertas por chapas de madeira

asfáltica.

(a) Desprezando a superposição das chapas de

madeira, qual a taxa de condução de calor através do

telhado, quando a temperatura no interior da

edificação for de 70 0F e no exterior 40 F ?

(b) Calcular a taxa de condução de calor se à

cobertura anterior forem superpostas 2 in de

isolamento especial para telhados.

19. A equação:

FY T

A

Fornece a tensão necessária para manter a

temperatura da barra constante à medida que a

temperatura varia. Mostre que se o comprimento

pudesse variar de ΔL quando sua temperatura varia de

ΔT, a tensão seria dada por:

0

LF A Y T

L

Onde:

F: tensão na barra.

L0: comprimento original da barra.

Y: Módulo de Young.

Α: coeficiente de dilatação linear.

20. Uma placa quadrada de aço de 10 cm de

lado é aquecida em uma forja de ferreiro até 1000C. Se

sua emissividade é e = 0.60, qual será a taxa total de

energia emitida por radiação ?

21 - Determine:

(a) As resistências térmicas do cobre, do aço

e a equivalente.

(b) O fluxo de calor através da barra de cobre

de seção quadrada da figura. A temperatura na

interface.

Dados: condutividade térmica:

cobre:Cu 385,0 J(s m°C)

-1

aço:Aço 50,2 J(s m°C)

-1

e

TA

dt

dQ

eR

A

22. – O espectro típico de uma lâmpada

fluorescente está indicado abaixo:

(a) Utilizando a Lei do deslocamento de

Wien:

max

2.898mm K

T

Ache a que temperatura corresponde ao máximo

comprimento de onda dessas lâmpadas. Observe que

o pico em comprimento de onda ocorre para essas

lâmpadas em torno de max = 305 nm.

(b) Aplicando a Lei de Stefan-Boltzman: 4H A e T

: constante de Stefan-Boltzmann. 2

4

85.6699 10 W m

K

Encontre a potência dissipada nessa temperatura,

assumindo área 20 cm2 e emissividade e = 1;

23. – As lâmpadas UV fluorescentes são

usualmente categorizadas como lâmpadas UVA,

UVB ou UVC, dependendo da região em que maior

parte de sua irradiação se situa. O espectro UV está

dividido dentro de três regiões:

Região UVA, 315 a 400 nanômetros;

Região UVB, 280 a 315 nanômetros;

Região UVC, abaixo de 280 nanômetros.

Complete a relação da tabela.

Dados: cf

; E h f 346,62 10h J s ;

c= 3.108m/s;

1240E eV

nm

Região

(0

A )

f

(Hz)

E

(eV)

UVA > 109 < 3 x 10

9 < 10

-5

UVB 10

9 -

106

10-5

- 0.01

UVC 10

6 -

7000

3 x 1012

- 4.3 x

1014

Visível

4.3 x

1014

-

7.5 x

1014

2 - 3

Page 28: Física Moderna Capítulo 4 - Radiação de corpo negro e ... · de correntes de convecção cde ar aquecido, que se movem para cima. Colocando-se em um dos lados do ... para a densidade

Física Moderna – Capítulo 4 - Radiação de corpo negro e Efeito Fotoelétrico

Prof. Dr. Cláudio S. Sartori

50

50

24. – Se colocarmos as barras indicadas numa

ligação em paralelo encontre a resistência térmica

equivalente e o fluxo total de calor.

Dados: condutividade térmica:

cobre:Cu 385,0 J(s m°C)

-1

aço:Aço 50,2 J(s m°C)

-1

e

TA

dt

dQ e

RA

25. – Explique o mecanismo das brisas

oceânicas.

26. – Determine o comprimento da barra

indicado para que o fluxo de calor seja de 250W.

27. Uma camada esférica de condutividade

térmica k tem um raio interno r1 e um raio externo r2.

A camada interna está a uma temperatura T1 e

a externa a uma temperatura T2. Mostre que a corrente

térmica é dada por:

1 2 2 1

2 1

4 k r r T T

r r

Solução:

k A T k A

dr dTe

24k rdr dT

2

4dr kdT

r

2 2

1 1

2

4R T

R T

dr kdT

r

2

2

1

1

1 4r R

T T

T Tr R

kT

r

2 1

2 1

1 1 4 kT T

R R

2 1

2 1

1 1 4 kT T

R R

2 12 1

1 2

4R R kT T

R R

1 22 1

2 1

4 k R RT T

R R

28. O raio interno a de uma casca cilíndrica

está mantido a uma temperatura Ta enquanto seu raio

externo b está a uma temperatura Tb. A casca

cilíndrica possui uma condutividade térmica

uniforme k. Mostre que o fluxo sobre a casca

cilíndrica é dada por:

2

ln

a bT TL k

b a

Page 29: Física Moderna Capítulo 4 - Radiação de corpo negro e ... · de correntes de convecção cde ar aquecido, que se movem para cima. Colocando-se em um dos lados do ... para a densidade

Física Moderna – Capítulo 4 - Radiação de corpo negro e Efeito Fotoelétrico

Prof. Dr. Cláudio S. Sartori

51

51

Solução:

k A T k A

dr dTe

2k r Ldr dT

2dr k LdT

r

2 b

a

Tb

a T

dr k LdT

r

2ln

b

a

r b T T

r a T T

k Lr T

2

ln ln b a

k Lb a T T

2

ln b a

k Lb a T T

2

ln

b aT TL k

b a

Fluxo de dentro para fora.

Fluxo de fora para dentro:

2

ln

b aT TL k

b a

2

ln

a bT TL k

b a

29. A seção de passageiros de uma vião a jato

tem a forma de uma tubulação cilíndrica com 35m de

comprimento e raio interno 2.50m. Sabe-se que a

espessura do tubo que compõe o avião é cerca de 6 cm

e tem uma condutividade térmica dada por 4.10-

5cal/(s.cm°C). A temperatura deve ser mantida dentro

em cerca de 25°C e fora do avião na altitude de

cruzeiro é cerca de -35°C. Que potência deve ser feita

para que se mantenha essa diferença de temperatura?

Solução:

Page 30: Física Moderna Capítulo 4 - Radiação de corpo negro e ... · de correntes de convecção cde ar aquecido, que se movem para cima. Colocando-se em um dos lados do ... para a densidade

Física Moderna – Capítulo 4 - Radiação de corpo negro e Efeito Fotoelétrico

Prof. Dr. Cláudio S. Sartori

52

52

O Efeito Fotoelétrico

Um dos fenômenos fundamentais para a

comprovação da teoria quântica é o efeito fotoelétrico

e o efeito Compton. Ambos envolvem a interação

entre a radiação eletromagnética e os elétrons

presentes na matéria.

A condução elétrica por meio de descarga gasosa

era um fenômeno comum de investigação na segunda

metade do século XIX. Um arranjo experimental típico

é mostrado na figura 1 abaixo.

Figura 1 – Tubo de raios catódicos. Aparato

experimental na investigação de corrente elétrica

gerada por descarga gasosa na segunda metade do

século XIX.

Uma alta voltagem era aplicada entre o

eletrodo negativo (cathode) e o eletrodo positivo

(anode), que causava um fluxo de corrente no tubo de

vidro, o qual era preenchido por certos gases a uma

baixa pressão. A corrente provoca camadas brilhantes

adjacentes de gás envoltas por camadas mais escuras,

cuja natureza depende da pressão do gás e do valor da

voltagem aplicada. Um conjunto complicado de

fenômenos é responsável pela condução elétrica e

brilho das camadas gasosas, cuja luz é emitida pelas

moléculas do gás causada pela sua ionização.

Quando a pressão no tubo é diminuída da

ordem de 0.01 mmHg ou menos, o gás para de

brilhar. Porém, o amperímetro ainda indica a

passagem de corrente elétrica. Outro fenômeno

observado é a presença de um brilho em S, na

superfície interior do tubo. O tamanho do spot S

(brilho) depende do tamanho e da localização dos

diafragmas D e D’. A posição da luminosidade do

spot pode ser influenciada pela presença de um forte

campo elétrico e magnético colocados numa região à

direita de D’, comprovando-se que as partículas que

atravessam a região estão eletricamente carregadas;

um sensor de posição indica que a carga das

partículas é negativa. Estas partículas, que

atravessam como um raio do cathode eram chamadas

de partículas de raios catódicos, ou simplesmente

raios catódicos e o aparato indicado na figura 1 é

chamado de tubo de raios catódicos.

Hoje sabemos que as partículas são emitidas

do cathodo como resultado do bombardeamento de

íons positivos do gás contidos no tubo.

Figura 2 – Aparato utilizado por J.J.

Thomson para encontrar a razão e/m do elétron.

Em 1897, Thomson mediu a razão entre a

carga e e a massa m do elétron com o aparato descrito

acima. As partículas de raios catódicos, de carga

negativa, atravessavam do cathodo C ao anodo A, o

qual era perfurado de um pequeno buraco, atuando

como o primeiro de dois diafragmas, D e D’, As

partículas passam por duas placas metálicas paralelas

de comprimento l e distanciadas de d. Finalmente

elas se chocam no final do tubo, produzindo uma

luminosidade S1.

A aplicação de uma voltagem V entre as

placas cria uma força que atua nas partículas numa

direção perpendicular à velocidade v, dada por:

e VF

d

Essa força produz uma aceleração

transversal dada por:

e Va

m d

Ela atua durante um tempo l/v durante o qual

as partículas estão atravessando a região entre as

placas. Após sair da região entre as placas, a deflexão

sofrida pelas partículas na direção transversal será:

21

2a t

Page 31: Física Moderna Capítulo 4 - Radiação de corpo negro e ... · de correntes de convecção cde ar aquecido, que se movem para cima. Colocando-se em um dos lados do ... para a densidade

Física Moderna – Capítulo 4 - Radiação de corpo negro e Efeito Fotoelétrico

Prof. Dr. Cláudio S. Sartori

53

53

21

2

e V l

m d v

A deflexão 1 2S S do spot luminoso é

amplificada, para pequenas deflexões, por um fator

2L/l, onde L é a distância do centro das placas até o

fim do tubo. 2

1 2

1 2

2

e V L lS S

m d l v

Se a deflexão 1 2S S é medida e a velocidade v

das partículas é conhecida, então o valor e/m será

descoberto.

Foi em 1886 e 1887 que Heinrich Hertz realizou

as experiências que pela primeira vez confirmaram a

existência de ondas eletromagnéticas e a teoria de

Maxwell sobre a propagação da luz. É um desses fatos

paradoxais e fascinantes na história da ciência que

Hertz tenha notado, no decorrer de suas experiências,

o efeito que Einstein mais tarde usou para contradizer

outros aspectos da teoria eletromagnética clássica.

Hertz descobriu que uma descarga elétrica entre dois

eletrodos ocorre mais facilmente quando se faz incidir

sobre um deles luz ultravioleta.

Lenard, seguindo alguns experimentos de

Hallwachs, mostrou logo em seguida que a luz

ultravioleta facilita a descarga ao fazer com que

elétrons sejam emitidos da superfície do cátodo. A

emissão de elétrons de uma superfície, devida à

incidência de luz sobre essa superfície, e' chamada

efeito fotoelétrico.

A figura 3 mostra um aparelho usado para estudar

o efeito fotoelétrico.

Figura 3 –

Um invólucro de vidro encerra o aparelho

em um ambiente no qual se faz vácuo. Luz

monocromática, incidente através de uma janela de

quartzo, cai sobre a placa de metal e libera elétrons

chamados fotoelétrons. Os elétrons podem ser

detectados sob a forma de uma corrente se forem

atraídos para o coletor metálico através de uma

diferença de potêncial V estabelecida. Um

amperímetro mede essa corrente fotoelétrica.

A curva da figura 4 e um gráfico da corrente da

fotoelétnca, em um aparelho como o figura 3 em

função da diferença de poteucial. Se V é muito

grande, a corrente foloelétrica atinge um certo valor

limite (ou de saturação) no qual todos os fotoelétrons

em emitidos por A são coletados por B.

Se o sinal de V é invertido, a corrente

fotoelétrica não cai imediatamente a zero, o que

sugere que os elétrons são emitidos de A com alguma

energia cinética. Alguns alcançarãocoletor B apesar

do campo elétrico opr-se ao seu movimento.

Entretanto, se essa diferença de potencial tornar-se

suficientemente grande, um valor V0 chamado

Page 32: Física Moderna Capítulo 4 - Radiação de corpo negro e ... · de correntes de convecção cde ar aquecido, que se movem para cima. Colocando-se em um dos lados do ... para a densidade

Física Moderna – Capítulo 4 - Radiação de corpo negro e Efeito Fotoelétrico

Prof. Dr. Cláudio S. Sartori

54

54

potencial limite ou de corteé atingido, e a corrente

fotoelétrica cai a zero. Essa diferença de potencial V0,

multiplicada pela carga do elétron –e, mede a energia

cinética máxima Kmax do mais rápido fotoelétron

emitido. Isto é:

max 0K e V

Experimentalmente nota-se que a quantidade

Kmax é independente da intensidade da luz incidente,

como é mostrado na figura 4 (a), na qual a intensidade

da luz foi reduzida à metade para se obter a curva I. Já

para freqüências diferentes, observa-se o mostrado na

figura 4 (b).

Figura 4 (a)

Figura 4 (b)

Há três aspectos do efeito fotoelétrico que

não podem ser explicados em termos da teoria

ondulatória clássica da luz:

1. A teoria ondulatória requer que a

amplitude do campo elétrico oscilante E da onda

luminosa cresça se a intensidade da luz for

aumentada. Já que a força aplicado ao elétron é e.E,

isso sugere que a energia cinética dos fotoelétrons

deveria crescer também ao se aumentar a intensidade

do feixe luminoso.Entretanto, como vimos na figura

anterior, Kmax = e.V0, independe da intensidade da

luz. Isto foi testado para variações de intensidade da

ordem de 107.

2. De acordo dom a teoria ondulatória, o

efeito fotoelétrico deveria ocorrer para qualquer

freqüência da luz, desde que fosse intensa o bastante

para dar a energia necessária à ejeção dos elétrons.

Entretanto, a figura 5 mostra que existe, para cada

superfície, um limiar de freqüências 0

característico. Para freqüências menores que 0, o

efeito fotoelétrico não ocorre, qualquer que seja a

intensidade da iluminação.

Figura 5 –

3. Se a energia requerida por um fotoelétron

é absorvida da onda incidente sobre a placa metálica,

a área de alvo efetiva para um elétron no metal é

limitada, e provavelmente não é muito maior que a de

um círculo de raio aproximadamente igual ao raio

atômico. Na teoria clássica, a energia luminosa está

uniformemente distribuída sobre a frente de onda.

Portanto, se a luz é suficientemente fraca, deveria

haver um intervalo de tempo mensurável, que vamos

estimar, entre o instante que a luz começa a incidir na

superfície e o instante da ejeção do fotoelétron.

Durante este intervalo, o elétron deveria estar

absorvendo energia do feixe, até que estivesse

acumulado o bastante para escapar. No entanto,

nenhum retardamento detectável foi jamais medido.

Esta discordância é particularmente

marcante quando a substância fotoelétrica for um

gás; nessas circunstâncias, mecanismos de absorção

coletivos podem ser ignorados e a energia do

fotoelétron emitido deve certamente ter sido extraída

do feixe luminoso por um único átomo ou molécula.

Page 33: Física Moderna Capítulo 4 - Radiação de corpo negro e ... · de correntes de convecção cde ar aquecido, que se movem para cima. Colocando-se em um dos lados do ... para a densidade

Física Moderna – Capítulo 4 - Radiação de corpo negro e Efeito Fotoelétrico

Prof. Dr. Cláudio S. Sartori

55

55

Exemplo 1 - Uma placa de potássio

é colocada a l m de uma fonte luminosa pouco intensa,

cuja potência é l W ; l joule/s. Suponha que um

fotoelétron ejetado possa ter coletado sua energia em

uma arca circular da placa, cujo raio r é, digamos, um

raio atómico: r = 1.10-10

m. A energia necessária para

remover um elétron da superfície de potássio é cerca

de 2.1 eV = 3.4.10-19

J (Joule).

(Um elétron-volt: l eV = l.6.10-19

J (joule) é a

energia ganha por um elétron, cuja carga é –e = -l.6.10-

19C (Coulomb), ao passar através de uma diferença de

potencial de l V (Volt).) Quanto tempo levaria o

elétron para absorver essa quantidade de energia da

fonte luminosa ?

Solução:

Suponha que a energia está uniformemente

distribuída sobre a frente de onda. A área de alvo é:

r2 = .10

-20m

2 . A área de uma esfera com r

de raio é 4r2 com o raio centrada na fonte. Se a fonte

irradia uniformemente em todas as direçõcs (isto é, se

a energia está uniformemente distribuída sobre frentes

de onda esféricas que se afastam da tome, de acordo

com a teoria clássica) a taxa por segundo a qual a

energia incide sobre o alvo é: 20

21

2

101 2.5 10

4 1R J s R J s

Supondo que toda essa potência é absorvida,

podemos calcular o tempo necessário para um eletron

para adquirir energia suficiente para escapar; e

encontramos: 19

2

21

3.4 101.4 10

2.5 10t t s

2mint

É claro que poderíamos diminuir o tempo

calculado supondo uma área efetiva maior. A hipótese

mais favorável, que a energia é transmitida por um

processo de ressonância da onda luminosa para o

elétron, leva a uma área de alvo igual a 2 onde é o

comprimento de onda da luz, mas ainda obteríamos

um retardamento finito que estaria bem dentro de

nossas possibilidades de medida experimental. (Para a

luz ultravioleta de t 10-2

s, por exemplo, entretanto,

não foi detectado nenhum retardamento em quaisquer

circunstâncias, as primeiras experiências fixando um

limite superior de 10-9

s para qualquer retardamento

possível!

A TEORIA QUÂNTICA DE EINSTEIN

SOBRE O EFEITO FOTOELÉTRICO

Em 1905 Einsten colocou em questão a teoria

clássica da luz, propôs uma nova teoria, e citou o

efeito fotoelétrico como uma aplicação que poderia

testar qual teoria estava correta. Isto aconteceu

vários anos antes do trabalho de Millikan, mas

Einstein foi influenciado pela experiência de Lenard.

Como já mencionamos, Planck originalmente

restringiu seu conceito de quantização de energia aos

elétrons nas paredes de um corpo negro. Planck

acreditava que a energia eletromagnética, uma vez

irradiada, se espalhava pelo espaço da mesma forma

que ondas de água se espalham na água. Em vez

disso, Einstein propôs que a energia radiante está

quantizada em pacotes concentrados, que mais tarde

vieram a ser chamados fótons.

Einstein argumentou que as experiências óticas

bem conhecidas de interferência e difração da

radiação eletromagnética haviam sido feitas apenas

em situações que envolviam um número muito

grande de fótons. Estas experiências fornecem

resultados que são médias do comportamento dos

fótons individuais. A presença dos fótons nessas

experiências não é mais aparente do que a presença

de gotas d'água isoladas em um jato de água de uma

mangueira de jardim, se o número de gotas é muito

grande. Evidentemente, as experiências de

interferência e difração mostram definitivamente que

os fotons não vão de onde são emitidos até onde são

absorvidos da mesma maneira simples que partículas

clássicas, como as gotas d'água, o fazem. Eles se

propagam como ondas clássicas, no sentido que

cálculos baseados neste tipo de propagação clássico

(em particular, na maneira que dua.s ondas se

superpõem. retorçando-se ou anulando-se

dependendo de sua fase relativa) explicam

corretamente em média como os fotons viajam.

Einstein não concentrou sua atenção na teoria

ondulatória familiar com que a luz se propaga, mas

sim na meneira corpuscular com que ela é emitida e

absorvida. Ele argumentou que a exigência de Planck

de que a energia das ondas eletromagneticas de

frequência irradiadas por uma fonte (por exemplo,

uma fonte de luz ultravioleta numa experiência

fotoelétrica) fosse apenas 0, ou h., ou 2 h.,..., n h..

implicava que no processo de ida de um estado de

energia n h.. para um estado de eneigia (n+1) h..;

a fonte emitia um pulso discreto de radiação

eletromagnetica com energia:

E h

Supôs também que no processo fotoelétrico um

fóton é completamente absorvido por um elétron no

fotocatodo.

Quando um eletron é emitido da superfície do

metal, sua energia cinética é:

K h w

onde h., é a energia do fóton incidente absorvido e

w é o trabalho necessário para remover o elétron do

Page 34: Física Moderna Capítulo 4 - Radiação de corpo negro e ... · de correntes de convecção cde ar aquecido, que se movem para cima. Colocando-se em um dos lados do ... para a densidade

Física Moderna – Capítulo 4 - Radiação de corpo negro e Efeito Fotoelétrico

Prof. Dr. Cláudio S. Sartori

56

56

metal. Esse trabalho é necessário para superar os

campos atrativos dos átomos na superfície e as perdas

de energia cinética devidas ás colisões internas do

elétron. Alguns elétrons estão mais fortemente ligados

do que outros, alguns perdem energia por colisões em

sua trajetória. No caso da ligação mais fraca e

nenhuma perda interna, o fotoelétron vai emergir com

a energia cinética máxima Kmax. Portanto:

max 0K h w

onde w0 é uma energia característica do metal chamada

função trabalho; é a energia mínima necessária para

um eletron atravessar a superfície do metal e escapar

ás forças atrativas que normalmente ligam o eletron ao

metal.

Vejamos agora como a hipótese de Einstein

resolve as três objeções levantadas contra a

interpretação ondulatória do efeito fotoelétrico.

Quanto á objeção 1 (o fato de que A' não

depende da intensidade da iluminação), a teoria do

fóton concorda integralmente com a experiência.

Dobrar a intensidade da luz meramente dobra o

número de fótons e portanto duplica a corrente

fotoelétrica: isto não muda a energia h. de cada fóton

ou a natureza do processo fotoelétrico descrita.

A objeção 2 (a existência de um limiar de

frequências) é removida imediatamente por

max 0K h w . Se Kmax é igual a zero, temos:

0h w

que significa que um fóton de frequência 0

tem exatamente a energia necessária para ejetar os

fotoelétrons, e nenhum excesso que possa aparecer

como energia cinética. Se a frequência for menor que

0 , os fótons, não importando quantos eles sejam

(isto é, quão intensa seja a iluminação), não terão

individualmente a energia necessária para ejetar

fotoelétrons.

A objeção 3 (a ausência de retardamento) é

eliminada pela hipótese do fóton, pois a energia

necessária é fornecida em pacotes concentrados. Ela

não se espalha uniformemente sobre uma área

extensa, como supusemos no exemplo 1, suposição

esta baseada na hipótese de que a teoria ondulatória

clássica seja verdadeira. Se houver luz incidindo

sobre o cátodo, haverá pelo menos um fóton que o

atinge; este fóton será imediatamente absorvido por

algum átomo, causando a imediata emissão de um

fotoelétron.

Vamos reescrever a equação fotoelétrica de

Einstein substituindo 0e V por maxK :

00

whV

e e

Portanto a teoria de Einstein prevê uma

relação linear entre o potencial limite V0 e a

frequência v, em completa concordância com

resultados experimentais. A inclinação da curva

experimental da figura deve ser h/e, portanto:

15

14 14

2.20 0.653.9 10

10 10 6 10

V VV s

Podemos determinar h multiplicando esta

razão pela carga eletrônica e. Portanto:

15 19 343.9 10 1.6 10 6.2 10h h J s

De uma análise muito mais cuidadosa desse

e de outros dados, inclusive dados obtidos com

superfícies de Lítio, Millikan achou o valor h =

6,57.10-34

j-s, com uma precisão de

aproximadamente 0,5%. Esta medida estava bem

próxima do valor de h deduzido da fórmula da

radiação de Planck. A concordância numérica das

duas determinações de h usando fenômenos e

teorias completamente diferentes, é notável.

Um valor moderno de h deduzido de

diversas experiências, é: 346.6262 10h J s

Citando Millikan: "O efeito fotoelétrico . . .

fornece uma prova independente da fornecida pela

radiação do corpo negro, da exatidao da hipótese

fundamental da teoria quântica, ou seja, a hipótese

da emissão descontinua ou explosiva da energia que

e absorvida das ondas pêlos constituintes

eletrônicos dos átomos. Ele materializa, por assim

dizer, a quantidade h descoberta por Planck em seu

estudo da radiação de corpo negro e, como nenhum

outro fenômeno, nos faz acreditar que o conceito

físico básico que esta por trás do trabalho de Planck

corresponde á realidade".

Exemplo 2 - Obter a função

trabalho para o sódio da figura.

4.39.1014

A interseçâo da linha reta na figura 2-3 com

o eixo horizontal é o limiar de frequências:

v0 = 4.39.1014

Hz. Substituindo em: 34 14

0 0 6.63 10 4.39 10h w w

19

0 2.92 10w J

19

0 19

12.92 10

1.6 10

eVw J

J

0 1.82w eV

O mesmo valor é obtido na figura para o módulo

da interseção da reta com o eixo vertical. Entretanto,

experiências modernas dão o valor mais alto 2,27 eV.

Para a maioria dos metais o valor da função

trabalho e da ordem de poucos elétrons-volt é igual

Page 35: Física Moderna Capítulo 4 - Radiação de corpo negro e ... · de correntes de convecção cde ar aquecido, que se movem para cima. Colocando-se em um dos lados do ... para a densidade

Física Moderna – Capítulo 4 - Radiação de corpo negro e Efeito Fotoelétrico

Prof. Dr. Cláudio S. Sartori

57

57

ao valor da função trabalho para a emissão

termoiônica desses metais. *

Exemplo 3 - A que taxa por unidade de área

os fótons atingem a placa metálica do exemplo 1?

Suponha que a luz seja monocromática, de

comprimento de onda = 5890 Å (luz amarela)

Solução: A taxa por unidade de área segundo a qual a

energia incide sobre uma placa metálica a l m de

distância de uma fonte luminosa de l W é:

2

2 2

18 10

4 1

Js J

R Rm s

17

25 10

eVR

m s

Cada fóton tem uma energia de:

cE h E h

834

7

3 106.63 10

5.89 10E

193.4 10E J

2.1E eV

Portanto a taxa R segundo a qual os fótons

atingem uma unidade de área da placa é:

17

2

1 fóton5 10

2.1

eVR

m s eV

17

2

fóton22.4 10R

m s

O efeito fotoelétrico pode ocorrer porque a

energia do fóton é exatamente igual a funçao trabalho

de 2,1 eV para a superfície de potássio. Note que se o

comprimento de onda for ligeiramente aumentado (isto

é, se v for ligeiramente diminuída) o efeito fotoelétrico

deixará de ocorrer, não importando quão grande seja a

taxa R.

Este exemplo sugere que a intensidade I da luz

pode ser interpretada como o produto de N, o número

de fótons por unidade de área por unidade de tempo,

por hv a energia de um único fóton. Vemos aqui que

mesmo a intensidades relativamente pequenas ( 10-1

W/m2 ) o número N é extremamente grande ( 10

17

fótons/(m2.s)) de modo que a energia de um único

fóton e muito pequena. Isto explica a extrema

granulosidade da radiação, e indica por que

normalmente e tão difícil detectá-la. Isto é análogo a

tentar detectar a estrutura atômica de uma porção de

matéria que, para a maioria dos fins, pode ser

considerada contínua, sua granulosidade sendo

revelada apenas em circunstancias especiais.

Em 1921 Einstein recebeu o Prêmio Nobel por ter

previsto teoricamente a lei do efeito fotoelétrico.

Antes que Millikan validasse experimentalmente

essa lei em 1914. l instem foi indicado para membro

da Academia Prussiana de Ciências por Planck e

outros. A atitude negativa inicial perante a hipótese"

do fóton é revelada por eles em sua declaração,

elogiando Einstein, na qual escreveram: "Em resumo,

podemos dizer que dificilmente haverá um grande

problema, dos quais a física moderna e tão rica, ao

qual Einstein não tenha dado uma importante

contribuição. Que ele tenha algumas vezes errado o

alvo em suas especulações, como por exemplo em

sua hipótese dos quanta de luz (fótons), não pode ser

realmente colocado contra ele, poi é impossível

introduzir ideias fundamentalmente novas, mesmo

nas ciências mais exalas, sem ocasionalmente correr

um risco".

Hoje a hipótese do fóton é usada em todo o

espectro eletromagnético, não apenas na região

visível. Pode-se dizer, por exemplo, que uma

cavidade de microondas contém fótons. Com = 10

cm, um comprimento de onda típico de microondas,

pode-se calcular, da mesma forma que anteriormente,

que a energia do fóton é 1,20 x 10 s eV. Esta energia

é muito pequena para ejetar fotoelétrons de

superfícies metálicas. Para raios X, ou para raios

tais como os que são emitidos por núcleos

radioativos, a energia do fóton pode ser de 106 eV, ou

mais. Estes fótons podem extrair de átomos pesados

elétrons fortemente ligados por energias da ordem de

10-5 eV. Os fótons na região visível do espectro

eletromagnético não tem energia suficiente para fazer

isto, e os fotoelétrons que eles ejetam são os

chamados elétrons de condução, que estão ligados ao

metal por energias de alguns elétrons-volt.

Observe que os fótons são absorvidos no

processo fotoelétrico. Isto requer que os elétrons

estejam ligados a átomos, ou sólidos, pois um elétron

completamente livre não pode absorver um fóton e

conservar simultaneamente a energia e o momento

relativísticos totais. Devemos ter um elétron ligado

para que as forças de ligação transmitam momento

para o átomo ou sólido.

Figura 6 - O espectro eletromagnético,

mostrando o comprimento de onda, a frequência e a

energia por fóton em escala logarítmica.

Page 36: Física Moderna Capítulo 4 - Radiação de corpo negro e ... · de correntes de convecção cde ar aquecido, que se movem para cima. Colocando-se em um dos lados do ... para a densidade

Física Moderna – Capítulo 4 - Radiação de corpo negro e Efeito Fotoelétrico

Prof. Dr. Cláudio S. Sartori

58

58

Devido à grande massa de um átomo, ou

sólido, comparada com a do elétron, o sistema pode

absorver uma grande quantidade de momento sem

adquirir uma quantidade significativa de energia.

Nossa equação para a energia fotoelétrica permanece

válida, e o efeito é possível apenas porque existe uma

partícula pesada que recua além do elétron ejetado. O

efeito fotoelétrico é uma maneira importante pela qual

fótons, com energias que vão até às dos raios X

(inclusive), são absorvidos pela matéria. A energias

mais altas, outros processos de absorção de fótons, que

logo discutiremos, tornam-se mais importantes.

Finalmente, devemos enfatizar aqui que no

modelo de Einstein um fóton de frequência v tem

exatamente a energia hv, e não múltiplos inteiros de

hv. Evidentemente, pode haver n fótons de frequência

v de modo que a energia nessa frequência seja nhv. Ao

tratar a radiação de uma cavidade de corpo negro com

o modelo de Einstein, lidamos com um ''gás de

fótons", pois a energia radiante está localizada no

espaço em pacotes em vez de estar espalhada em

ondas estacionárias. Anos depois de Planck ter

deduzido sua fórmula para a radiação de cavidade,

Bose e Einstein obtiveram a mesma fórmula baseados

em um gás de fótons.

Figura 7 – Efeito fotoelétrico.

Figura 8 – Efeito fotoelétrico. Montagem

experimental.

Figura 9 – Aplicação do Efeito

fotoelétrico: Fotomultiplicadoras.

Page 37: Física Moderna Capítulo 4 - Radiação de corpo negro e ... · de correntes de convecção cde ar aquecido, que se movem para cima. Colocando-se em um dos lados do ... para a densidade

Física Moderna – Capítulo 4 - Radiação de corpo negro e Efeito Fotoelétrico

Prof. Dr. Cláudio S. Sartori

59

59

Tabela- Função trabalho para diversos

elementos.

Elemento Função trabalho (eV)

Alumínio 4.3

Carbono 5.0

Cobre 4.7

Ouro 5.1

Níquel 5.1

Silício 4.8

Prata 2.7

Sódio 2.7

Exemplo 4 - Um filme de silício torna-se

um bom condutor elétrico quando iluminado com

fótons de energia de 1.14 eV ou superior (Este

comportamento é denominado de fotocondutividade.)

Qual é o comprimento de onda correspondente?

Solução:

h c h cE

E

34 86.6262 10 3 10

E

251.98786 10m

E J

Como 19 181 1.6 10 1 6.25 10eV J J eV

251.98786 10m

E J

1241.875nm

E eV

1241.875

1.14nm

1090nm

Este comprimento de onda está na região

do infravermelho do EE. A energia mínima de 1.14

eV corresponde ao comprimento de onda máximo

que produz fotocondutividade no silício, portanto

todos os fótons da luz visível também produzirão

fotocondutividade pois possuem comprimento de

onda menores do que esse limite e energias mais

elevadas do que aquela.

Exemplo 5 – Uma experiência do

efeito fotolétrico. Realizando uma experiência do

efeito fotoelétrico com a luz de determinado

comprimento de onda, você verifica que é necessário

uma diferença de potencial invertida de 1.25 V para

anular a corrente. Determine:

(a) a energia cinética máxima;

(b) a velocidade máxima dos fotoelétrons

emitidos.

Solução:

(a)19

max 0 max 1.6 10 1.25K e V K

max 1.25K eV

(b) 2

max max max 0

1

2eK m v K e V

19

maxmax max 31

2 2 2 10

9.11 10e

Kv v

m

5

max 6.63 10m

vs

Essa velocidade equivale a 1/500 da

velocidade da luz; logo, podemos utilizar a equação

não relativística para a energia cinética.

Exemplo 6 – Experiência para determinar

e e h. Para um certo material do catodo na

experiência do efeito fotoelétrico, verifica-se um

potencial de corte de 1.0 V para uma luz de

comprimento de onda de 600 nm, 2.0 V para 400 nm

e 3.0 V para 300 nm. Determine a função trabalho

para esse material e a constante de Planck.

Solução:

Como 0e V h f

Page 38: Física Moderna Capítulo 4 - Radiação de corpo negro e ... · de correntes de convecção cde ar aquecido, que se movem para cima. Colocando-se em um dos lados do ... para a densidade

Física Moderna – Capítulo 4 - Radiação de corpo negro e Efeito Fotoelétrico

Prof. Dr. Cláudio S. Sartori

60

60

0

hV f

e e

A partir dessa forma,vemos que a inclinação

da reta é igual a h/e e a interseção com o eixo vertical

(correspondente a f = 0) ocorre no ponto (0,-/e). As

frequências obtidas pela relação c f são:

(nm)

f (Hz)

600 815

9

3 100.50 10

600 10

cf f f

400 150.75 10

300 151.0 10

Do gráfico:

1e

(interseção vertical).

191 1.6 10eV J

Inclinação:

190 0

15

3.0 ( 1.0) .4.0 10

1 10 0

V V J s

f f C

190 4.0 10Vh h

e f e

19 19 194.0 10 4.0 10 1.6 10h e h 346.4 10h J s

Exemplo 7 – Fótons de uma rádio FM.

Uma estação de rádio transmite ondas com frequência

89.3 MHz com potência total igual a 43.0 kW.

(a) Qual é o módulo do momento linear de

cada fóton?

(b) Quantos fótons ela emite por segundo?

Solução: (a) Energia de cada fóton:

34 66.6262 10 89.3 10E h f E

265.92 10E J

Cada fóton tem um momento linear dado por: 26

8

5.92 10

3 10

Ep p

c

341.97 10p kg m s

(b) A estação emite 43.103J a cada segundo.

A taxa de emissão de fótons é: 3

26

43 10

5.92 10f

J sn

J fóton

297.26 10fn fótons s

Como é muito grande o número de fótons que

deixam a emissora de rádio a cada segundo, os saltos

de energia desses pequenos pacotes individuais não

são percebidos, fazendo com que a energia irradiada

pareça um fluxo contínuo.

Exemplo 8 – Luz do Sol. A

superfície do Sol possui uma temperatura

aproximadamente igual a 5800 K. Com boa

aproximação, podemos considerá-la um corpo negro.

(a) Qual é o comprimento de onda max que

fornece a intensidade do pico?

(b) Qual é a potência total irradiada por

unidade de área?

Solução: Usando a Lei do deslocamento de Wien:

32.9 10 2.9m m

m K mm K

T T

32.9 10500

5800m m

m Knm

(b) Usando a Lei de Stefan-Boltzman:

14P A e T

4PI T

A

8 45.67 10 5800I

7

26.42 10

WI

m

264.2

MWI

m

Esse valor enorme indica a intensidade na

superfície do Sol. Quando a potência irradiada atinge

a Terra, a intensidade cai para 1400W/m2 porque ela

se espalha para fora do Sol e atinge a área muito

grande de uma superfície esférica cujo raio é a

distância entre a Terra e o Sol.

Exemplo 9 – Calcule a intensidade da luz

emitida da superfície do Sol no intervalo entre 600nm

e 605.5 nm. Lei da radiação de Planck:

2

5

2

1h c

k T

h cI

e

Solução: Como resultado exato:

2

1

600.5 2

5600.0

2

1h c

k T

h cI I d d

e

Essa integral não pode ser calculada com

base em funções familiares. , logo, aproximamos a

área pelo produto da altura medida no comprimento

Page 39: Física Moderna Capítulo 4 - Radiação de corpo negro e ... · de correntes de convecção cde ar aquecido, que se movem para cima. Colocando-se em um dos lados do ... para a densidade

Física Moderna – Capítulo 4 - Radiação de corpo negro e Efeito Fotoelétrico

Prof. Dr. Cláudio S. Sartori

61

61

de onda médio = 602.5 nm vezes a largura do

intervalo ( = 5.0 nm). Inicialmente, calculamos: 34 8

7 23

1

6.62 10 . 3 10

6.025 10 1.381 10 5800

ms

JK

J sh c

k T m K

1

4.116h c

k T

2

5

2

1h c

k T

h cI

e

34 8

7 23

234 8

7

6.62 10 3105

7 6.025 10 1.38110 5800

2 6.62 10 3 106.025 10

6.025 10 1

I m

e

7 13

36.025 10 7.81 10

WI m

m

A intensidade no intervalo de 5 nm entre os

limites de 600.0 nm e 605.0 nm é, aproximadamente

igual a:

13 97.81 10 5 10I

5

23.9 10

WI

m

20.39

MWI

m

Ou seja, 0.6% da intensidade luminosa total

proveniente do Sol se encontra entre os limites de

600.0 nm e 605.0 nm

Exemplo 10 – Mostre que, usando a Lei da

radiação de Planck:

2

5

2

1h c

k T

h cI

e

2 5 4

4

2 3

50 0

2 2

151

h c

k T

h c kI d d T

c he

E que, a constante de Stefan-Boltzmann é

dada por: 5 4

2 3

2

15

k

c h

8

2 45.6705 10

W

m K

Com:

Constante de Boltzmann: 231.381 10 J

Kk

Constante de Planck: 346.62 10 .h J s

Velocidade da luz no vácuo:

83 10 ms

c

Solução:

50

1

1A

x

dx

x e

Essa integral vc não acha nem a pau!!

Sugestão:

http://integrals.wolfram.com

2

c cd d

2 cd d

c

2

2 2

c cd d d d

c

2

5 2

0

2

1h

k T

h c cd

ce

3

2

0

2

1h

k T

hd

ce

Use agora: 43

0

1 2

1 240x

xdx

e

Page 40: Física Moderna Capítulo 4 - Radiação de corpo negro e ... · de correntes de convecção cde ar aquecido, que se movem para cima. Colocando-se em um dos lados do ... para a densidade

Física Moderna – Capítulo 4 - Radiação de corpo negro e Efeito Fotoelétrico

Prof. Dr. Cláudio S. Sartori

62

62